[obm-l] Re: [obm-l] Questões legais - AVENTURA ????

2013-04-10 Por tôpico Rogerio Ponce
Ola' pessoal,

No problema1, observemos que:
- duas casas pares, quando combinadas, geram uma diferenca tambem par, e
a quantidade de numeros impares nao se altera.
- uma casa impar quando combinada com uma casa par, gera uma diferenca
impar, e a quantidade de impares nao se altera.
- duas casas impares, quando combinadas, geram uma diferenca par, e a
quantidade de impares diminui de 2.

Como existem 999 impares no quadro negro, e a quantidade de impares diminui
somente em pulos de 2, e' impossivel obtermos um numero par no final do
processo.


No problema2, existem 11 estradas.
Suponhamos que haja uma reta que corte todas as estradas.
Ao percorrermos todas as cidades, a partir da cidade 1 (e voltando para
ela), ao atravessarmos a tal reta, mudariamos de lado (em relacao a tal
reta) 11 vezes, ou seja, terminariamos o percurso do lado oposto ao lado da
partida, o que e' absurdo.
Logo nao existe a tal reta.

[]'s
Rogerio Ponce



2013/4/10 Robério Alves prof_robe...@yahoo.com.br


  *PROBLEMA1*
 Os números naturais de 1 até 1998 são escritos em um imenso quadro negro.
 Em seguida, um aluno apaga dois quaisquer colocando no lugar sua diferença
 (não negativa). Depois de muitas operações, um único número ficará escrito
 no quadro. É possível que esse número seja zero?
 ** **
 ** **
 *PROBLEMA 2*
 Em uma ilha plana existem 11 cidades numeradas de 1 a 11. Estradas retas
 ligam 1 a 2, 2 a 3, 3 a 4, ..., 10 a 11 e 11 a 1. É possível que uma reta
 corte todas as estradas?


 AVENTURA 






 --
 Esta mensagem foi verificada pelo sistema de antivírus e
 acredita-se estar livre de perigo.


-- 
Esta mensagem foi verificada pelo sistema de antivírus e
 acredita-se estar livre de perigo.



[obm-l] Re: [obm-l] Questões interessantes (na minha opinião)

2013-03-02 Por tôpico Bernardo Freitas Paulo da Costa
2013/3/1 Artur Costa Steiner steinerar...@gmail.com:
 1) suponhamos que exista uma função f tal que, para todo real x, tenhamos 
 f(f(x)) = ax^2 + bx + c, a não nulo, b e c reais. Mostre que (b +1)(b - 3) = 
 4ac.
Esse eu ainda tenho que pensar com cuidado. A primeira coisa é reduzir
a g(g(x)) = x^2 + c, mas eu ainda não sei fazer o caso c  0.

 2) seja (a_n) uma sequência de reais e (p_n) uma sequência de pesos 
 positivos. Seja (s_n) a sequência das médias ponderadas de (a_n) com relação 
 os pesos p_n. Mostre que, se Soma p_n divergir, então

 liminf s_n = liminf a_n = limsup a_n = limsup s_n

 É bem fácil mostrar que, se Soma p_n convergir, as desigualadas da direita e 
 da esquerda não têm que valer.
Curioso... Eu diria que s_n é uma combinação convexa dos a_n, logo s_N
= min(a_n, n=1..N) e portanto min(s_N, N=1..k) = min(min(a_n,
n=1..N), N=1..k) = min(a_n, n=1..k). Claro que tem que fazer do outro
lado (no infinito, não no 1) mas eu diria que liminf a_n = liminf
s_n. Mais tarde tento enviar uma prova dessa soma de Césaro.

 3) Seja f uma função definida em um intervalo I de R (suponhamos aberto, para 
 facilitar) e com valores em R. Suponhamos que, em cada ponto de I, as 4 
 derivadas de Dini de f existam e sejam finitas. Mostre que existe um 
 subintervalo de I no qual f é Lipschitz.
 Isto é mais fácil de mostrar se supusermos diferenciabilidade cheia em todo o 
 I. Mas, de fato, basta a existência das 4 derivadas de Dini.
Diferenciabilidade cheia = full differentiability = diferenciável no
sentido usual ? (Nunca fiz nada com derivadas de Dini)

-- 
Bernardo Freitas Paulo da Costa

-- 
Esta mensagem foi verificada pelo sistema de antivírus e
 acredita-se estar livre de perigo.


=
Instruções para entrar na lista, sair da lista e usar a lista em
http://www.mat.puc-rio.br/~obmlistas/obm-l.html
=


Re: [obm-l] Re: [obm-l] Questões interessantes (na minha opinião)

2013-03-02 Por tôpico Artur Costa Steiner
Para o caso da condição de Lipschitz, supondo que f seja diferenciável em I, me 
ocorreu uma vez o seguinte

1) f' é, conforme se sabe, o limite de uma sequência de funções contínuas.

2) Como R é um espaço de Baire, para toda sequência g_n de funções contínuas em 
um intervalo I  que convirja para uma função g, existe um subintervalo no qual 
as g_n são uniformemente limitadas por algum M  0. Logo, g é limitada por M 
neste subintervalo. 

3) de (2) segue-se haver um subintervalo de I no qual f' é limitada por algum M 
 0. Logo, neste subintervalo f' é Lipschitz e M é uma constante da condição de 
Lipschitz.


Uma vez eu mostrei esta prova para algumas pessoas e não gostaram. Paciência, 
não se pode agradar a todos. No caso, não agradei ninguém. Alguns disseram que 
estava errado, porque sabidamente diferenciabilidade não implica que a função 
seja localmente Lipschitz. Mas a condição que eu citei não é ser localmente 
Lipschitz, é mais fraca do que isso. 

Depois vim a saber que para, haver o subintervalo em que f seja Lipschitz, 
basta que em cada ponto de I as 4 derivadas de Dini de f sejam finitas. Me 
enrolei nesta prova, mas acho que tenho uma por contradição.

Aliás, nos complexos há uma conclusão interessante. Se f é inteira, então f é 
Lipschitz em todo conjunto limitado do plano complexo.

Abraços


Artur Costa Steiner

Em 02/03/2013, às 17:50, Bernardo Freitas Paulo da Costa 
bernardo...@gmail.com escreveu:

 2013/3/1 Artur Costa Steiner steinerar...@gmail.com:
 1) suponhamos que exista uma função f tal que, para todo real x, tenhamos 
 f(f(x)) = ax^2 + bx + c, a não nulo, b e c reais. Mostre que (b +1)(b - 3) 
 = 4ac.
 Esse eu ainda tenho que pensar com cuidado. A primeira coisa é reduzir
 a g(g(x)) = x^2 + c, mas eu ainda não sei fazer o caso c  0.
 
 2) seja (a_n) uma sequência de reais e (p_n) uma sequência de pesos 
 positivos. Seja (s_n) a sequência das médias ponderadas de (a_n) com relação 
 os pesos p_n. Mostre que, se Soma p_n divergir, então
 
 liminf s_n = liminf a_n = limsup a_n = limsup s_n
 
 É bem fácil mostrar que, se Soma p_n convergir, as desigualadas da direita e 
 da esquerda não têm que valer.
 Curioso... Eu diria que s_n é uma combinação convexa dos a_n, logo s_N
 = min(a_n, n=1..N) e portanto min(s_N, N=1..k) = min(min(a_n,
 n=1..N), N=1..k) = min(a_n, n=1..k). Claro que tem que fazer do outro
 lado (no infinito, não no 1) mas eu diria que liminf a_n = liminf
 s_n. Mais tarde tento enviar uma prova dessa soma de Césaro.
 
 3) Seja f uma função definida em um intervalo I de R (suponhamos aberto, 
 para facilitar) e com valores em R. Suponhamos que, em cada ponto de I, as 4 
 derivadas de Dini de f existam e sejam finitas. Mostre que existe um 
 subintervalo de I no qual f é Lipschitz.
 Isto é mais fácil de mostrar se supusermos diferenciabilidade cheia em todo 
 o I. Mas, de fato, basta a existência das 4 derivadas de Dini.
 Diferenciabilidade cheia = full differentiability = diferenciável no
 sentido usual ? (Nunca fiz nada com derivadas de Dini)
 
 -- 
 Bernardo Freitas Paulo da Costa
 
 -- 
 Esta mensagem foi verificada pelo sistema de antivírus e
 acredita-se estar livre de perigo.
 
 
 =
 Instruções para entrar na lista, sair da lista e usar a lista em
 http://www.mat.puc-rio.br/~obmlistas/obm-l.html
 =

-- 
Esta mensagem foi verificada pelo sistema de antiv�rus e
 acredita-se estar livre de perigo.


=
Instru��es para entrar na lista, sair da lista e usar a lista em
http://www.mat.puc-rio.br/~obmlistas/obm-l.html
=


[obm-l] Re: [obm-l] Questões lógicas

2011-02-26 Por tôpico Marcelo Costa
o próximo é 200, todos os números começam com D
Abraços

Em 25 de fevereiro de 2011 11:00, Marco Bivar Jr.
marco.bi...@gmail.comescreveu:

 Duas questões lógicas para os colegas deleitarem-se:

 1. Qual o número X na sequência: 2, 10, 12, 16, 17, 18, 19, X, ...?

 2. Um fazendeiro decidiu doar sua fazenda para apenas um de seus dois
 filhos, que a teria logo após a sua morte. Ele decidiu que deveriam
 circundar a fazenda numa volta a cavalo, e que o dono do cavalo mais lento,
 este ficaria com a fazenda. Haviam dois cavalos pretos no estábulo, e cada
 um pertencia a um filho. Os filhos, então, pegaram um cavalo, e
 apressaram-se para completar a volta em primeiro. Por que?

 --
 Marco Bivar Jr.



[obm-l] Re: [obm-l] Re: [obm-l] Questões lógicas

2011-02-26 Por tôpico Marco Bivar Jr.
A questão 1 foi tirada da lousa em sala de aula na faculdade. A número 2 é
uma piada contada por um amigo. Portanto, não pensei 2 vezes ao postá-las na
lista. Quanto à questão 1, se eu falo português, então X=200. A questão 2
não merece atenção pois vocês viram quantas interpretações ela pode dar.
Deleitarem-se, Se deleitarem...acho que é questão de estética; ou seja,
você escolhe. Havia por Haviam...talvez você esteja certo.

--- Marco Bivar Jr.

Em 26 de fevereiro de 2011 07:17, Marcelo Costa mat.mo...@gmail.comescreveu:

 o próximo é 200, todos os números começam com D
 Abraços

 Em 25 de fevereiro de 2011 11:00, Marco Bivar Jr. 
 marco.bi...@gmail.comescreveu:

 Duas questões lógicas para os colegas deleitarem-se:

 1. Qual o número X na sequência: 2, 10, 12, 16, 17, 18, 19, X, ...?

 2. Um fazendeiro decidiu doar sua fazenda para apenas um de seus dois
 filhos, que a teria logo após a sua morte. Ele decidiu que deveriam
 circundar a fazenda numa volta a cavalo, e que o dono do cavalo mais lento,
 este ficaria com a fazenda. Haviam dois cavalos pretos no estábulo, e cada
 um pertencia a um filho. Os filhos, então, pegaram um cavalo, e
 apressaram-se para completar a volta em primeiro. Por que?


 --
 Marco Bivar Jr.





-- 
Marco Bivar Jr.


[obm-l] RE: [obm-l] Questões lógicas

2011-02-25 Por tôpico Artur Steiner

1. Não está parecendo um problema matemático. Na realidade, nenhuma sequência 
fica definida conhecendo-se um número finito de seus termos. Há uma infinidade 
de possibilidades poara se determinar o próximo termo. Vc pode, por exemplo, 
ajustar um polinômio aos pontos dados e estimar os outros com base neste 
polinômio. Isto é possível para uma infinidade de polinômios de gruas maior ou 
igual ao número de pontos -1.

Aqui, vemos que todos os números, quando grafados em Português, começam com d. 
Se esta for sa lei de formação, o próximo número é duzentos. Em outra´s 
línguas, como o Inglês, isto não vale. 

 

2. Por que cada filho pegu o cavalo do irmão.

Ou então, ambos eram pessoas altruístas, cada um pegou seu próprio cavalo e 
procurou chegar em primeiro lugar para que o irmão ficasse com a fazenda.

 

Artur

 

 

 

5 Feb 2011 11:00:01 -0300
Subject: [obm-l] Questões lógicas
From: marco.bi...@gmail.com
To: obm-l@mat.puc-rio.br

Duas questões lógicas para os colegas deleitarem-se:




1. Qual o número X na sequência: 2, 10, 12, 16, 17, 18, 19, X, ...?


2. Um fazendeiro decidiu doar sua fazenda para apenas um de seus dois filhos, 
que a teria logo após a sua morte. Ele decidiu que deveriam circundar a fazenda 
numa volta a cavalo, e que o dono do cavalo mais lento, este ficaria com a 
fazenda. Haviam dois cavalos pretos no estábulo, e cada um pertencia a um 
filho. Os filhos, então, pegaram um cavalo, e apressaram-se para completar a 
volta em primeiro. Por que?
-- 
Marco Bivar Jr.
  

[obm-l] Re: [obm-l] Questões lógicas

2011-02-25 Por tôpico João Luís Gomes Guimarães
1. 200

2. Se cada um tenciona vencer a corrida, significa que cada irmão pegou o 
cavalo que pertencia ao outro

From: Marco Bivar Jr. 
Sent: Friday, February 25, 2011 11:00 AM
To: obm-l@mat.puc-rio.br 
Subject: [obm-l] Questões lógicas

Duas questões lógicas para os colegas deleitarem-se:


1. Qual o número X na sequência: 2, 10, 12, 16, 17, 18, 19, X, ...?

2. Um fazendeiro decidiu doar sua fazenda para apenas um de seus dois filhos, 
que a teria logo após a sua morte. Ele decidiu que deveriam circundar a fazenda 
numa volta a cavalo, e que o dono do cavalo mais lento, este ficaria com a 
fazenda. Haviam dois cavalos pretos no estábulo, e cada um pertencia a um 
filho. Os filhos, então, pegaram um cavalo, e apressaram-se para completar a 
volta em primeiro. Por que?

-- 
Marco Bivar Jr.


[obm-l] Re: [obm-l] RE: [obm-l] Questões lógicas

2011-02-25 Por tôpico Bernardo Freitas Paulo da Costa
2011/2/25 Artur Steiner artur_stei...@hotmail.com:
 2. Por que cada filho pegu o cavalo do irmão.
2. também não parece um problema matemático.

 Ou então, ambos eram pessoas altruístas, cada um pegou seu próprio cavalo e
 procurou chegar em primeiro lugar para que o irmão ficasse com a fazenda.
Ou entao a fazenda estava endividada até o pescoço... e ninguém queria
ficar com ela.

E para defender a língua pátria: para os colegas *se* deleitarem,
preposição exige próclise. E Havia dois cavalos, pelamordedeus.

 Artur
-- 
Bernardo Freitas Paulo da Costa

=
Instruções para entrar na lista, sair da lista e usar a lista em
http://www.mat.puc-rio.br/~obmlistas/obm-l.html
=


[obm-l] Re: [obm-l] Questões resolvam pra ver - ajuda

2010-10-05 Por tôpico Vinícius
Quem pode me ajudar urgente?
Cada cartela de uma coleção é formada por seis quadrados coloridos,
justapostos
como indica a figura abaixo.
 __
I___I___I___I
I___I___I___I

Em cada cartela, dois quadrados foram coloridos de azul,
dois de verde e dois de rosa. A coleção apresenta todas as
possibilidades de distribuição dessas cores nas cartelas nas
condições citadas e não existem cartelas com a mesma
distribuição de cores. Retirando-se ao acaso uma cartela da
coleção, a probabilidade de que somente uma coluna
apresente os quadrados de mesma cor é de:


[obm-l] RE: [obm-l] Questões simples

2009-10-21 Por tôpico marcone augusto araújo borges

2) Mariana tem x filhos,Paula,3x e Carlos tem 6x.Total:10x,um múltiplo de 
10.Dos números apresentados,o único múltiplo de 10 é 10.Não é isso?
 


Date: Wed, 21 Oct 2009 09:06:43 -0700
From: diog...@yahoo.com.br
Subject: [obm-l] Questões simples
To: obm-l@mat.puc-rio.br





VocÊ pode ajudar?
 
01. Durante quanto tempo em um dia a marcação de um relógio digital, no modo 24 
horas, indica o número de horas superior ao número de minutos?
 
02. Mariana, Carlos e Paula são irmãos e cada um deles tem uma quantidade 
diferentes de filhos. Carlos tem o dobro do número de filhos de Paula e Paula 
tem o triplo do número de filhos de Mariana. Qual o total de filhos desses 
irmãos? 8/10/12/14 ou 16 filhos?
 
03. Paulo, Pedro e Plínio são amigos e fazem aniversário no mesmo dia. Sabe-se 
que eles nasceram no Rio de Janeiro, Belo Horizonte e São Paulo, em anos 
alternados. Paulo nasceu dois anos antes de Plínio. Pedro é o mais velho e não 
nasceu no Rio de Janeiro. Plínio tem 18 anos e é mineiro. O que posso afirmar?
a. Pedro tem 20 anos;
b. O mais novo é carioca;
c. Paulo é Paulista;
d. O carioca tem 20 anos;
e. O Carioca é mais novo que  o mineiro.
 
Agradeço pela atenção,
 
Diogo FN
http://diogofn.6te.net
 


Veja quais são os assuntos do momento no Yahoo! + Buscados: Top 10 - 
Celebridades - Música - Esportes 
_
Você sabia que pode acessar o Messenger direto do seu Hotmail? Descubra como!
http://www.microsoft.com/brasil/windows/windowslive/products/tutoriais.aspx

[obm-l] RE: [obm-l] RE: [obm -l] Re: [obm-l] Re: [obm-l] Questões de Combinatória. (ajuda)

2009-04-28 Por tôpico Jordan Piva

Tudo bem, isso acontece. Espero ter ajudado tb.

Abrcs

From: joao_maldona...@hotmail.com
To: obm-l@mat.puc-rio.br
Subject: [obm-l] RE: [obm-l] Re: [obm-l] Re: [obm-l] Questões de Combinatória. 
(ajuda)
Date: Sun, 26 Apr 2009 10:17:09 -0300








Obrigado Rafael e Jordan, foi uma completa falta de desatenção mesmo, acho que 
eu estava com pressa indo para o show do cézar menotti e fabiano que nem 
percebi os erros (só pra você ver na q.4, contei a solução 2^6.3^6 e não contei 
2^6 nem 3^6). Desculpe pelos erros Vinícius, não vai acontecer de novo.

Abraço

From: rafael.a...@gmail.com
Date: Sat, 25 Apr 2009 13:42:05 -0300
Subject: [obm-l] Re: [obm-l] Re: [obm-l] Questões de Combinatória. (ajuda)
To: obm-l@mat.puc-rio.br

Existem mais possibilidades a serem removidas na questão 4... Sabemos que se um 
número é, simultaneamente, um quadrado perfeito e um cubo perfeito, então ele é 
uma sexta potência.

Logo, basta remover todas as sextas potências de 1 a 100=10^6, ou seja, 
remover 10:



Então temos: 1000 + 100 - 10 = 1090.

Como o problema pergunta quantos números NÃO são quadrados nem cubos, a 
resposta é 100-1090 = 998910.

2009/4/24 Joao Maldonado joao_maldonad...@yahoo.com.br




Ola Vinícius, aí vai...

1.) O número não vai começar com 0 e o número deve começar com 53, 54, 56, 57, 
6 ou 7.
53, 54, 56 ou 57 - 4.6!/3!
6 ou 7 - 2.7!/3!
Total = 6.5.4.(4+2.7) = 120.18 = 2160 possibilidades.



2.)
6! = 720 posibilidades (porém nesse resultado o mesmo cubo pode ser
encontrado de 6 maneiras somente fazendo uma rotação de um outro cubo),
caso contrário seriam 6!/6 = 5! = 120 possibilidades

3.) a) n!
b)
Caso a minha interpretação esteja correta como voxê colocou a conjunção
e ao invés da ou no final da frase, não poderia acontecer as 3
coisas SIMULTANEAMENTE, ou seja, o primeiro lugar ser o número 1, o
segundo o número 2 e o terceiro o número 4 é uma possibilidade válida.
Consequentementeteríamos (n-3)! possibilidades da corrida terminar com
1-2-3, assim a resposta é: n! - (n-3)!

4.) Esse quatro é mais legalzinho.
OK, quadrado perfeito: 1² = 1 e 1000² = 100 - Teremos 1000 quadrados 
perfeitos.
cubos perfeitos - 1³ = 1 e 100³ = 100 - Teremos 100 cubos perfeitos.


Toda
quarta potência é um quadrado então consequentemente podemos ignorar
esta opção. Temos que tirar os casos em que x² = y³ - ou seja, x =
a1^6k.a2^6k...an^6k e y = b1^6k.b2^6k...bn^6k para todo ai e bi primos
(além da solução x=1).
Temos no máximo x ou y produto das potências de 2 primos pois 2^6.3^6.5^6  
100
Temos k = 1 pois: 2^12.3^12  100
Possibilidades: (1) ; 2^6.3^6 = (46656) ; 2^6.5^6 = (100)


Total = 1000 + 100 - 3 = 1097 possiblidades.

Abraço,

João

--- Em sex, 24/4/09, Vinícius pvni...@gmail.com escreveu:



De: Vinícius pvni...@gmail.com
Assunto: [obm-l] Questões de Combinatória. (ajuda)
Para: obm-l@mat.puc-rio.br


Data: Sexta-feira, 24 de Abril de 2009, 16:21

1.  Quantos números inteiros de cinco algarismos distintos e maiores do que 
53.000 podemser formados com os algarismos 0, 1, 2, 3, 4, 5, 6 e 7?


2.  De quantos modos se pode pintar um cubo, usando seis cores fixas
 distintas, sendo cada
face de uma cor?
3.  Em uma corrida há n participantes. Antes de a corrida começar, cada 
participante recebeum número entre 1 e n.a) De quantas maneiras diferentes os 
participantes podem terminar a corrida?


b) De quantas maneiras o 1o lugar NÃO é o participante número 1, o 2o lugar NÃO 
é oparticipante número 2 e o 3o lugar NÃO é o participante número 3?
4. Quantos inteiros entre 1 e 100, inclusive, não são quadrados perfeitos, 
nem cubos


perfeitos,nem quartas potências perfeitas?




  Veja quais são os assuntos do momento no Yahoo! + Buscados: Top 10 - 
Celebridades - Música - Esportes



-- 
Rafael

Quer saber qual produto Windows Live combina melhor com o seu perfil? Clique 
aqui e descubra!
_
Descubra seu lado desconhecido com o novo Windows Live!
http://www.windowslive.com.br

[obm-l] RE: [obm-l] Questões de Combinatória. (a juda)

2009-04-27 Por tôpico João Maldonado

Ola Vinícius, aí vai...

1.) O número não vai começar com 0 e o número deve começar com 53, 54, 56, 57, 
6 ou 7.
53, 54, 56 ou 57 - 4.6!/3!
6 ou 7 - 2.7!/3!
Total = 6.5.4.(4+2.7) = 120.18 = 2160 possibilidades.

2.) 6! = 720 posibilidades (porém nesse resultado o mesmo cubo pode ser 
encontrado de 6 maneiras somente fazendo uma rotação de um outro cubo), caso 
contrário seriam 6!/6 = 5! = 120 possibilidades

3.) a) n!
b) Caso a minha interpretação esteja correta como voxê colocou a conjunção e 
ao invés da ou no final da frase, não poderia acontecer as 3 coisas 
SIMULTANEAMENTE, ou seja, o primeiro lugar ser o número 1, o segundo o número 2 
e o terceiro o número 4 é uma possibilidade válida. Consequentementeteríamos 
(n-3)! possibilidades da corrida terminar com 1-2-3, assim a resposta é: n! - 
(n-3)!

4.) Esse quatro é mais legalzinho.
OK, quadrado perfeito: 1² = 1 e 1000² = 100 - Teremos 1000 quadrados 
perfeitos.
cubos perfeitos - 1³ = 1 e 100³ = 100 - Teremos 100 cubos perfeitos.
Toda quarta potência é um quadrado então consequentemente podemos ignorar esta 
opção. Temos que tirar os casos em que x² = y³ - ou seja, x = 
a1^6k.a2^6k...an^6k e y = b1^6k.b2^6k...bn^6k para todo ai e bi primos (além da 
solução x=1).
Temos no máximo x ou y produto das potências de 2 primos pois 2^6.3^6.5^6  
100
Temos k = 1 pois: 2^12.3^12  100
Possibilidades: (1) ; 2^6.3^6 = (46656) ; 2^6.5^6 = (100)
Total = 1000 + 100 - 3 = 1097 possiblidades.

Abraço,

João


Date: Fri, 24 Apr 2009 13:21:05 -0300
Subject: [obm-l] Questões de Combinatória. (ajuda)
From: pvni...@gmail.com
To: obm-l@mat.puc-rio.br

1.  Quantos números inteiros de cinco algarismos distintos e maiores do que 
53.000 podemser formados com os algarismos 0, 1, 2, 3, 4, 5, 6 e 7?
2.  De quantos modos se pode pintar um cubo, usando seis cores fixas distintas, 
sendo cada
face de uma cor?
3.  Em uma corrida há n participantes. Antes de a corrida começar, cada 
participante recebeum número entre 1 e n.a) De quantas maneiras diferentes os 
participantes podem terminar a corrida?
b) De quantas maneiras o 1o lugar NÃO é o participante número 1, o 2o lugar NÃO 
é oparticipante número 2 e o 3o lugar NÃO é o participante número 3?
4. Quantos inteiros entre 1 e 100, inclusive, não são quadrados perfeitos, 
nem cubos
perfeitos,nem quartas potências perfeitas?
_
Faça já uma busa e ganhe um wink do Messenger. Está esperando o que? É grátis!
http://www.ibud.com.br/

[obm-l] Re: [obm-l] RE: [obm-l ] Questões de Combin atória . (ajuda)

2009-04-27 Por tôpico Vinícius
Que isso João, os erros acontecem. Muito obrigado pela força galera!grande
abraço


[obm-l] RE: [obm-l] Re: [obm -l] Questões de Comb inatória. (ajuda)

2009-04-25 Por tôpico Jordan Piva

Bem Joao ha um erro na sua solucao p/ questao 2, veja que na realidade teríamos 
inicialmente  6!=720 formas porém você tem que descontar as rotacoes do cubo. 
Fixe uma face (como se estivesse segurando o cubo com uma face em sua direção). 
Com esta face fixa voltada para você quantas rotações podemos fazer? quatro. 
Como podemos fixar qualquer uma das 6 faces temos um total de 6x4=24 casos 
repetidos a serem desconsiderados assim:

 

Resp.: 720/24=30

 

 


Date: Fri, 24 Apr 2009 16:20:33 -0700
From: joao_maldonad...@yahoo.com.br
Subject: [obm-l] Re: [obm-l] Questões de Combinatória. (ajuda)
To: obm-l@mat.puc-rio.br





Ola Vinícius, aí vai...

1.) O número não vai começar com 0 e o número deve começar com 53, 54, 56, 57, 
6 ou 7.
53, 54, 56 ou 57 - 4.6!/3!
6 ou 7 - 2.7!/3!
Total = 6.5.4.(4+2.7) = 120.18 = 2160 possibilidades.

2.) 6! = 720 posibilidades (porém nesse resultado o mesmo cubo pode ser 
encontrado de 6 maneiras somente fazendo uma rotação de um outro cubo), caso 
contrário seriam 6!/6 = 5! = 120 possibilidades

3.) a) n!
b) Caso a minha interpretação esteja correta como voxê colocou a conjunção e 
ao invés da ou no final da frase, não poderia acontecer as 3 coisas 
SIMULTANEAMENTE, ou seja, o primeiro lugar ser o número 1, o segundo o número 2 
e o terceiro o número 4 é uma possibilidade válida. Consequentementeteríamos 
(n-3)! possibilidades da corrida terminar com 1-2-3, assim a resposta é: n! - 
(n-3)!

4.) Esse quatro é mais legalzinho.
OK, quadrado perfeito: 1² = 1 e 1000² = 100 - Teremos 1000 quadrados 
perfeitos.
cubos perfeitos - 1³ = 1 e 100³ = 100 - Teremos 100 cubos perfeitos.
Toda quarta potência é um quadrado então consequentemente podemos ignorar esta 
opção. Temos que tirar os casos em que x² = y³ - ou seja, x = 
a1^6k.a2^6k...an^6k e y = b1^6k.b2^6k...bn^6k para todo ai e bi primos (além da 
solução x=1).
Temos no máximo x ou y produto das potências de 2 primos pois 2^6.3^6.5^6  
100
Temos k = 1 pois: 2^12.3^12  100
Possibilidades: (1) ; 2^6.3^6 = (46656) ; 2^6.5^6 = (100)
Total = 1000 + 100 - 3 = 1097 possiblidades.

Abraço,

João

--- Em sex, 24/4/09, Vinícius pvni...@gmail.com escreveu:


De: Vinícius pvni...@gmail.com
Assunto: [obm-l] Questões de Combinatória. (ajuda)
Para: obm-l@mat.puc-rio.br
Data: Sexta-feira, 24 de Abril de 2009, 16:21



1.  Quantos números inteiros de cinco algarismos distintos e maiores do que 
53.000 podem
ser formados com os algarismos 0, 1, 2, 3, 4, 5, 6 e 7?


2.  De quantos modos se pode pintar um cubo, usando seis cores fixas distintas, 
sendo cada
face de uma cor?


3.  Em uma corrida há n participantes. Antes de a corrida começar, cada 
participante recebe
um número entre 1 e n.
a) De quantas maneiras diferentes os participantes podem terminar a corrida?
b) De quantas maneiras o 1o lugar NÃO é o participante número 1, o 2o lugar NÃO 
é o
participante número 2 e o 3o lugar NÃO é o participante número 3?


4. Quantos inteiros entre 1 e 100, inclusive, não são quadrados perfeitos, 
nem cubos
perfeitos,nem quartas potências perfeitas?


Veja quais são os assuntos do momento no Yahoo! + Buscados: Top 10 - 
Celebridades - Música - Esportes
_
Messenger 2009: Instale já!
http://download.live.com

[obm-l] Re: [obm-l] Re: [obm-l] Questões de Combinatória. (ajuda)

2009-04-25 Por tôpico Rafael Ando
Existem mais possibilidades a serem removidas na questão 4... Sabemos que se
um número é, simultaneamente, um quadrado perfeito e um cubo perfeito, então
ele é uma sexta potência.

Logo, basta remover todas as sextas potências de 1 a 100=10^6, ou seja,
remover 10:

Então temos: 1000 + 100 - 10 = 1090.

Como o problema pergunta quantos números NÃO são quadrados nem cubos, a
resposta é 100-1090 = 998910.

2009/4/24 Joao Maldonado joao_maldonad...@yahoo.com.br

 Ola Vinícius, aí vai...

 1.) O número não vai começar com 0 e o número deve começar com 53, 54, 56,
 57, 6 ou 7.
 53, 54, 56 ou 57 - 4.6!/3!
 6 ou 7 - 2.7!/3!
 Total = 6.5.4.(4+2.7) = 120.18 = 2160 possibilidades.

 2.) 6! = 720 posibilidades (porém nesse resultado o mesmo cubo pode ser
 encontrado de 6 maneiras somente fazendo uma rotação de um outro cubo), caso
 contrário seriam 6!/6 = 5! = 120 possibilidades

 3.) a) n!
 b) Caso a minha interpretação esteja correta como voxê colocou a conjunção
 e ao invés da ou no final da frase, não poderia acontecer as 3 coisas
 SIMULTANEAMENTE, ou seja, o primeiro lugar ser o número 1, o segundo o
 número 2 e o terceiro o número 4 é uma possibilidade válida.
 Consequentementeteríamos (n-3)! possibilidades da corrida terminar com
 1-2-3, assim a resposta é: n! - (n-3)!

 4.) Esse quatro é mais legalzinho.
 OK, quadrado perfeito: 1² = 1 e 1000² = 100 - Teremos 1000 quadrados
 perfeitos.
 cubos perfeitos - 1³ = 1 e 100³ = 100 - Teremos 100 cubos perfeitos.
 Toda quarta potência é um quadrado então consequentemente podemos ignorar
 esta opção. Temos que tirar os casos em que x² = y³ - ou seja, x =
 a1^6k.a2^6k...an^6k e y = b1^6k.b2^6k...bn^6k para todo ai e bi primos (além
 da solução x=1).
 Temos no máximo x ou y produto das potências de 2 primos pois 2^6.3^6.5^6 
 100
 Temos k = 1 pois: 2^12.3^12  100
 Possibilidades: (1) ; 2^6.3^6 = (46656) ; 2^6.5^6 = (100)
 Total = 1000 + 100 - 3 = 1097 possiblidades.

 Abraço,

 João

 --- Em *sex, 24/4/09, Vinícius pvni...@gmail.com* escreveu:


 De: Vinícius pvni...@gmail.com
 Assunto: [obm-l] Questões de Combinatória. (ajuda)
 Para: obm-l@mat.puc-rio.br
 Data: Sexta-feira, 24 de Abril de 2009, 16:21


 1.  Quantos números inteiros de cinco algarismos distintos e maiores do que
 53.000 podem
 ser formados com os algarismos 0, 1, 2, 3, 4, 5, 6 e 7?

 2.  De quantos modos se pode pintar um cubo, usando seis cores fixas
 distintas, sendo cada
 face de uma cor?

 3.  Em uma corrida há n participantes. Antes de a corrida começar, cada
 participante recebe
 um número entre 1 e n.
 a) De quantas maneiras diferentes os participantes podem terminar a
 corrida?
 b) De quantas maneiras o 1o lugar NÃO é o participante número 1, o 2o lugar
 NÃO é o
 participante número 2 e o 3o lugar NÃO é o participante número 3?

 4. Quantos inteiros entre 1 e 100, inclusive, não são quadrados
 perfeitos, nem cubos
 perfeitos,nem quartas potências perfeitas?


 --
 Veja quais são os assuntos do momento no Yahoo! + Buscados: Top 
 10http://br.rd.yahoo.com/mail/taglines/mail/*http://br.maisbuscados.yahoo.com/-
 Celebridadeshttp://br.rd.yahoo.com/mail/taglines/mail/*http://br.maisbuscados.yahoo.com/celebridades/-
 Músicahttp://br.rd.yahoo.com/mail/taglines/mail/*http://br.maisbuscados.yahoo.com/m%C3%BAsica/-
 Esporteshttp://br.rd.yahoo.com/mail/taglines/mail/*http://br.maisbuscados.yahoo.com/esportes/




-- 
Rafael


[obm-l] Re: [obm-l] Questões de Combinatória. (ajuda )

2009-04-24 Por tôpico Joao Maldonado
Ola Vinícius, aí vai...

1.) O número não vai começar com 0 e o número deve começar com 53, 54, 56, 57, 
6 ou 7.
53, 54, 56 ou 57 - 4.6!/3!
6 ou 7 - 2.7!/3!
Total = 6.5.4.(4+2.7) = 120.18 = 2160 possibilidades.

2.)
6! = 720 posibilidades (porém nesse resultado o mesmo cubo pode ser
encontrado de 6 maneiras somente fazendo uma rotação de um outro cubo),
caso contrário seriam 6!/6 = 5! = 120 possibilidades

3.) a) n!
b)
Caso a minha interpretação esteja correta como voxê colocou a conjunção
e ao invés da ou no final da frase, não poderia acontecer as 3
coisas SIMULTANEAMENTE, ou seja, o primeiro lugar ser o número 1, o
segundo o número 2 e o terceiro o número 4 é uma possibilidade válida.
Consequentementeteríamos (n-3)! possibilidades da corrida terminar com
1-2-3, assim a resposta é: n! - (n-3)!

4.) Esse quatro é mais legalzinho.
OK, quadrado perfeito: 1² = 1 e 1000² = 100 - Teremos 1000 quadrados 
perfeitos.
cubos perfeitos - 1³ = 1 e 100³ = 100 - Teremos 100 cubos perfeitos.
Toda
quarta potência é um quadrado então consequentemente podemos ignorar
esta opção. Temos que tirar os casos em que x² = y³ - ou seja, x =
a1^6k.a2^6k...an^6k e y = b1^6k.b2^6k...bn^6k para todo ai e bi primos
(além da solução x=1).
Temos no máximo x ou y produto das potências de 2 primos pois 2^6.3^6.5^6  
100
Temos k = 1 pois: 2^12.3^12  100
Possibilidades: (1) ; 2^6.3^6 = (46656) ; 2^6.5^6 = (100)
Total = 1000 + 100 - 3 = 1097 possiblidades.

Abraço,

João

--- Em sex, 24/4/09, Vinícius pvni...@gmail.com escreveu:

De: Vinícius pvni...@gmail.com
Assunto: [obm-l] Questões de Combinatória. (ajuda)
Para: obm-l@mat.puc-rio.br
Data: Sexta-feira, 24 de Abril de 2009, 16:21

1.  Quantos números inteiros de cinco algarismos distintos e maiores do que 
53.000 podemser formados com os algarismos 0, 1, 2, 3, 4, 5, 6 e 7?
2.  De quantos modos se pode pintar um cubo, usando seis cores fixas distintas, 
sendo cada
face de uma cor?
3.  Em uma corrida há n participantes. Antes de a corrida começar, cada 
participante recebeum número entre 1 e n.a) De quantas maneiras diferentes os 
participantes podem terminar a corrida?
b) De quantas maneiras o 1o lugar NÃO é o participante número 1, o 2o lugar NÃO 
é oparticipante número 2 e o 3o lugar NÃO é o participante número 3?
4. Quantos inteiros entre 1 e 100, inclusive, não são quadrados perfeitos, 
nem cubos
perfeitos,nem quartas potências perfeitas?



  Veja quais são os assuntos do momento no Yahoo! +Buscados
http://br.maisbuscados.yahoo.com

[obm-l] Re: [obm-l] questões LIVRO DE ANÁLISE DO ELON V1

2009-03-31 Por tôpico Denisson
Questão 4:

= sE A intersecção B complementar é vazio entao para todo x pertencente a A
implica que x não pertence a B complementar o que implica que x pertence a
B, logo A está contido em B.

= Se A está contido em B então para todo x pertencente a A então x pertence
a B, logo x não pertence a B complementar. isto quer dizer que A intersecção
com B complementar é vazio.


questão 5 - Basta tomar A intersecção com B não vazia e tomar C como um
conjunto que não tem elementos comuns nem com A nem com B.



2009/3/30 Robÿe9rio Alves prof_robe...@yahoo.com.br



PÁGINA 29 DO LIVRO DE ANÁLISE DO ELON

 04) Dados A, B está contido  em E, prove que A está contido em B se,
 somente se, A ∩ Complementar de B =  Ø

 Questão 5) Dê exemplos de conjuntos A, B, C tais que ( A U B ) ∩ C ≠ A U (
 B ∩C)


 QUESTÃO 8)  Prove que A = B se, e somente se, ( A ∩ Complementar de B ) U
 ( Complementar de A ∩ B ) = Ø





  --
 Veja quais são os assuntos do momento no Yahoo! + Buscados: Top 
 10http://br.rd.yahoo.com/mail/taglines/mail/*http://br.maisbuscados.yahoo.com/-
 Celebridadeshttp://br.rd.yahoo.com/mail/taglines/mail/*http://br.maisbuscados.yahoo.com/celebridades/-
 Músicahttp://br.rd.yahoo.com/mail/taglines/mail/*http://br.maisbuscados.yahoo.com/m%C3%BAsica/-
 Esporteshttp://br.rd.yahoo.com/mail/taglines/mail/*http://br.maisbuscados.yahoo.com/esportes/




-- 
Denisson


[obm-l] Re: [obm-l] questões LIVRO DE ANÁLISE DO ELON

2009-03-30 Por tôpico Marcelo Salhab Brogliato
Olá Robério,

vou tentar fazer a 4..
ida) Se A C B, entao para todo a E A, a E B, logo a \E compl(B), assim, nao
existe a E A tq a E compl(B), logo: A inter compl(B) = {}.
volta) Se A inter compl(B) = {}, entao nao existe a E A tq a E compl(B),
assim para todo a E A temos que a \E compl(B), logo a E compl(compl(B)) = B,
logo, A C B.

espero ter ajudado.. e espero que esse monte de letras seja legível! hehehe

abraços,
Salhab



2009/3/30 Robÿe9rio Alves prof_robe...@yahoo.com.br



PÁGINA 29 DO LIVRO DE ANÁLISE DO ELON

 04) Dados A, B está contido  em E, prove que A está contido em B se,
 somente se, A ∩ Complementar de B =  Ø

 Questão 5) Dê exemplos de conjuntos A, B, C tais que ( A U B ) ∩ C ≠ A U (
 B ∩C)


 QUESTÃO 8)  Prove que A = B se, e somente se, ( A ∩ Complementar de B ) U
 ( Complementar de A ∩ B ) = Ø





 --
 Veja quais são os assuntos do momento no Yahoo! + Buscados: Top 
 10http://br.rd.yahoo.com/mail/taglines/mail/*http://br.maisbuscados.yahoo.com/-
 Celebridadeshttp://br.rd.yahoo.com/mail/taglines/mail/*http://br.maisbuscados.yahoo.com/celebridades/-
 Músicahttp://br.rd.yahoo.com/mail/taglines/mail/*http://br.maisbuscados.yahoo.com/m%C3%BAsica/-
 Esporteshttp://br.rd.yahoo.com/mail/taglines/mail/*http://br.maisbuscados.yahoo.com/esportes/



[obm-l] Re: [obm-l] questões topologia da reta

2009-01-27 Por tôpico Rafael Assis
Para resolver o segundo, basta ver como é feito o conjunto de Cantor:
Na primeira iteração, retira-se o terço do meio do intervalo [0,1], ou seja,
um intervalo de comprimento um terço.
Na segunda iteração retiram-se dois intervalos de comprimento um terço de um
terço, isto é, dois nonos.
Note que os comprimentos dos intervalos omitidos( você pode fazer mais
iterações para observar isto) a cada iteração forma uma progressão
geométrica infinita cujo primeiro termo é um terço e a razão 2 terços.Logo,
a soma
da serie formada por tais termos deve ser 1.

Saudações,

Rafael


[obm-l] Re: [obm-l] Questões de Geometria e Teoria dos N úmeros

2008-08-16 Por tôpico luiz silva
Rafael,
 
A mesma correção de enunciado é valida para a 2a. questão sobre teoria dos 
números.
 
Abs
Felipe

--- Em qui, 14/8/08, Rafael Ando [EMAIL PROTECTED] escreveu:

De: Rafael Ando [EMAIL PROTECTED]
Assunto: Re: [obm-l] Questões de Geometria e Teoria dos Números
Para: obm-l@mat.puc-rio.br
Data: Quinta-feira, 14 de Agosto de 2008, 10:03



Ah, uma coisa... note que Prove que a equação diofantina x2 + y2 = zn possui 
infinitas soluções inteiras NAO eh a mesma coisa que qqer potência de n pode 
ser representada com a soma de 2 quadrados - mesmo que vc tivesse dito 
qualquer potencia de z ou qualquer n-esima potencia)... 3^3 = 27, por 
exemplo, nao pode ser escrito como soma de quadrados... Acredito mesmo que a 
maioria dos pares (z,n) nao tenha solucao!
Acho ainda que o enunciado nao esta correto. nao seria algo do tipo: Prove 
que, para todo n, x2 + y2 = zn possui infinitas soluções inteiras? Pra mim nao 
eh a mesma coisa pois se n for uma das variaveis eu posso escolher x e y 
qualquer, n=1 e z = x^2 + y^2...


2008/8/14 luiz silva [EMAIL PROTECTED]









Olá Martin/Pessoal,
 
Qdo entrei para esta lista, enviei um email com algmas questões que tinha 
criado, para análise de vcs. Porém, acho estranho não ter , ainda, visualisado 
esta msg na minha caixa de entrada, o que me faz pensar que deve ter havido 
algum problema com o envio desta msg.Dessa foram, estou enviando novamente.
 
Caso esta msg já tenha ido para a lista, favor desconsiderar este email.
 
Se possível, gostaria de um feedback de vcs, com relação a dificuldade destes 
problemas.

 

 Geometria
 
1) Considere um ângulo  90 BÂC 180, com ABAC. Com centro em B e raio AB, 
trace um arco de circunferência (AD) de 120. Com centro em C, e raio AC, 
trace outro arco de circunferência (AE) de 120, de modo que DÂEBÂC(a figura é 
côncava). Marque, sobre o arco de circunferência AD um ponto F, tal que DF= l12 
( L minúsculo..rs ). Marque, sobre o arco de circunferência AE o ponto G, tal 
que GE= l12(referente ao arco em que se está marcando o ponto). Trace o 
segmento de reta que une GF, marcando seu ponto M, médio. Marque um ponto H, 
externo ao polígono ABDFGEC, tal que o ângulo D^HE = 120 e HD=HE. O segmento HB 
intercepta o arco AD em P, e o segmento MB intercepta o arco AD em Q. Calcule o 
ângulo PÂQ.  
 
2) Considere o quadrilátero PABC, onde PA=PB=PC, se o ângulo B^PC= 22, calcule 
CÂB.
 
3) Dado o hexágono regular ABCDEF, , um ponto P exterior a este hexagono, com 
PÂF=150 e PA=PF, um ponto Q exterior a este hexágono, tal que F^QE=150 e QF=QE. 
Os segmentos QB e PE conrtam-se em L; QB e PD em M; QC e PD em N e QC e PE em 
O. Calcule L^NO
 
4) Dado um triângulo escaleno qualquer ABC. Trace a altura AH (H pé da altura). 
A partir de H, trace uma perpendicular a AB que intercepte este lado num ponto 
D e uma perpendicular ao lado AC, que intercepte este lado em um ponto E. A 
partir de D, trace uma perpendicular a AC, que intercepte este lado em F e de E 
trace uma perpendicular a AB, que intercepte o segmento DF em G. Sabendo 
que BC=a, AB=c e AC=b e AH=h, calcule a área do quedrilátero DHEG em funçaõ dos 
lados do triângulo e da altura h. 
 
Teoria dos Números
 
1) Prove que a equação diofantina x2 + y2 = zn possui infinitas soluções 
inteiras (ou seja, que qqer potência de n pode ser representada com a soma de 2 
quadrados).
 
2) Prove que a equãção diofantina  xn + yn = zn+1 possui infinitas soluções 
inteiras 



Novos endereços, o Yahoo! que você conhece. Crie um email novo com a sua cara 
@ymail.com ou @rocketmail.com.


-- 
Rafael



  Novos endereços, o Yahoo! que você conhece. Crie um email novo com a sua 
cara @ymail.com ou @rocketmail.com.
http://br.new.mail.yahoo.com/addresses

[obm-l] Re: [obm-l] Questões de Geometria e Teoria dos N úmeros

2008-08-14 Por tôpico luiz silva
Ola Rafael,
 
Vc está correto. O enunciado deveria ser : 
 
Prove que a equação diofantina x2 + y2 = zn possui soluções inteiras 
(x,y,z) para qualquer n natural. 
 
Abs
Felipe

--- Em qui, 14/8/08, Rafael Ando [EMAIL PROTECTED] escreveu:

De: Rafael Ando [EMAIL PROTECTED]
Assunto: Re: [obm-l] Questões de Geometria e Teoria dos Números
Para: obm-l@mat.puc-rio.br
Data: Quinta-feira, 14 de Agosto de 2008, 10:03



Ah, uma coisa... note que Prove que a equação diofantina x2 + y2 = zn possui 
infinitas soluções inteiras NAO eh a mesma coisa que qqer potência de n pode 
ser representada com a soma de 2 quadrados - mesmo que vc tivesse dito 
qualquer potencia de z ou qualquer n-esima potencia)... 3^3 = 27, por 
exemplo, nao pode ser escrito como soma de quadrados... Acredito mesmo que a 
maioria dos pares (z,n) nao tenha solucao!
Acho ainda que o enunciado nao esta correto. nao seria algo do tipo: Prove 
que, para todo n, x2 + y2 = zn possui infinitas soluções inteiras? Pra mim nao 
eh a mesma coisa pois se n for uma das variaveis eu posso escolher x e y 
qualquer, n=1 e z = x^2 + y^2...


2008/8/14 luiz silva [EMAIL PROTECTED]









Olá Martin/Pessoal,
 
Qdo entrei para esta lista, enviei um email com algmas questões que tinha 
criado, para análise de vcs. Porém, acho estranho não ter , ainda, visualisado 
esta msg na minha caixa de entrada, o que me faz pensar que deve ter havido 
algum problema com o envio desta msg.Dessa foram, estou enviando novamente.
 
Caso esta msg já tenha ido para a lista, favor desconsiderar este email.
 
Se possível, gostaria de um feedback de vcs, com relação a dificuldade destes 
problemas.

 

 Geometria
 
1) Considere um ângulo  90 BÂC 180, com ABAC. Com centro em B e raio AB, 
trace um arco de circunferência (AD) de 120. Com centro em C, e raio AC, 
trace outro arco de circunferência (AE) de 120, de modo que DÂEBÂC(a figura é 
côncava). Marque, sobre o arco de circunferência AD um ponto F, tal que DF= l12 
( L minúsculo..rs ). Marque, sobre o arco de circunferência AE o ponto G, tal 
que GE= l12(referente ao arco em que se está marcando o ponto). Trace o 
segmento de reta que une GF, marcando seu ponto M, médio. Marque um ponto H, 
externo ao polígono ABDFGEC, tal que o ângulo D^HE = 120 e HD=HE. O segmento HB 
intercepta o arco AD em P, e o segmento MB intercepta o arco AD em Q. Calcule o 
ângulo PÂQ.  
 
2) Considere o quadrilátero PABC, onde PA=PB=PC, se o ângulo B^PC= 22, calcule 
CÂB.
 
3) Dado o hexágono regular ABCDEF, , um ponto P exterior a este hexagono, com 
PÂF=150 e PA=PF, um ponto Q exterior a este hexágono, tal que F^QE=150 e QF=QE. 
Os segmentos QB e PE conrtam-se em L; QB e PD em M; QC e PD em N e QC e PE em 
O. Calcule L^NO
 
4) Dado um triângulo escaleno qualquer ABC. Trace a altura AH (H pé da altura). 
A partir de H, trace uma perpendicular a AB que intercepte este lado num ponto 
D e uma perpendicular ao lado AC, que intercepte este lado em um ponto E. A 
partir de D, trace uma perpendicular a AC, que intercepte este lado em F e de E 
trace uma perpendicular a AB, que intercepte o segmento DF em G. Sabendo 
que BC=a, AB=c e AC=b e AH=h, calcule a área do quedrilátero DHEG em funçaõ dos 
lados do triângulo e da altura h. 
 
Teoria dos Números
 
1) Prove que a equação diofantina x2 + y2 = zn possui infinitas soluções 
inteiras (ou seja, que qqer potência de n pode ser representada com a soma de 2 
quadrados).
 
2) Prove que a equãção diofantina  xn + yn = zn+1 possui infinitas soluções 
inteiras 



Novos endereços, o Yahoo! que você conhece. Crie um email novo com a sua cara 
@ymail.com ou @rocketmail.com.


-- 
Rafael



  Novos endereços, o Yahoo! que você conhece. Crie um email novo com a sua 
cara @ymail.com ou @rocketmail.com.
http://br.new.mail.yahoo.com/addresses

[obm-l] Re: [obm-l] Re: [obm-l] Questões da OBM

2007-10-30 Por tôpico barola
É, tem razão.
Deixei passar tal argumento..
Entendi agora.

Obrigada.
Abraçosss..
  - Original Message - 
  From: Fetofs Ashu 
  To: obm-l@mat.puc-rio.br 
  Sent: Monday, October 29, 2007 8:20 PM
  Subject: Re: [obm-l] Re: [obm-l] Questões da OBM


  Bárbara,

  Lembra do meu ponto 1? Se você tem um grupo (x, y, z, w), só há um termo que 
pode vir antes desses quatro termos, quaisquer sejam eles.

  Provar o ponto 1 é trivial, já que precisamos de um valor congruente a 
w-(z+y+x) mod 10 e não há dois termos dos possíveis membros da sequência (0 a 
9) que têm o mesmo valor mod 10. 

  Mesmo se você não entender de aritmética modular, o ponto 1 é muito 
intuitivo. Pegue alguns grupos (x, y, z, w) quaisquer e veja se você consegue 
achar dois termos que podem vir antes desses. Você vai logo se cansar, já que 
não tem jeito :) 

  (não continue até entender o que eu disse até agora)

  Logo a sequência (i, j, k, l) não pode vir depois de um h e depois de um 
l ao mesmo tempo (claro, se considerarmos que h é diferente de l). Concluímos 
que a situação proposta é impossível. 

  Note que para rejeitar um ciclo de período indefinido, precisamos do ponto 2. 
Como cada grupo (c, d, e, f) só tem um termo que pode antecedê-lo (chamaremos 
de b), o grupo (b, c, d, e) também só tem um termo que pode antecedê-lo (um 
termo a qualquer). Logo, cada termo só pode vir de uma sequência definida 
(por exemplo, os números 1, 1, 1, 3 só podem vir depois de 0, 8, 9, 2, 9, 0, 
etc.) 

  Fernando Oliveira




  On 10/29/07, [EMAIL PROTECTED]  [EMAIL PROTECTED] wrote:
Desculpe a ignorância, mas porque não podemos pensar que o ciclo seja 
com um período parcial?
Assim: a,b,c,d,e,f,g,h,i,j,k,l,i,j,k,l,i,j,k,l,i,j,k,l,.
Acho que sua solução está certa, só faltou provar que não dá certo para 
esse caso, concorda?

Mesmo assim, você, o Nicolau e todos os grandes alunos e mestres desta 
lista tem me ensinado muito!
Obrigada mesmo!

- Original Message - 
  From: Fetofs Ashu 
  To: obm-l@mat.puc-rio.br 
  Sent: Monday, October 29, 2007 11:28 AM
  Subject: Re: [obm-l] Questões da OBM


  Salhab e Bárbara,

  1) Vamos andar para trás. Se você tem um grupo (x, y, z, w), só há um 
termo que pode vir antes desses quatro termos, quaisquer sejam eles.
  2) Continuando o processo de 1) temos que todo grupo só pode ser obtido 
através de uma sequência definida.
  3) Um grupo deve se repetir, pois o número de grupos possíveis é finito.
  4) Como na primeira vez que esse grupo apareceu (1, 2, 3, 4) fazia parte 
da sequência, deve fazer na segunda vez também, já que a sequência é única.

  Fernando Oliveira 



[obm-l] Re: [obm-l] Questões da OBM

2007-10-29 Por tôpico barola
 Oi Shine!
 Achei realmente muito interessante a sua solução para o problema das 
pilhas!
No entanto, não entendi, assim como o Salhab, como 1,2,3,4 vai aparecer de 
novo!
Henrique, meu nome é Bárbara sim. Só que eu tive que colocar um apelido 
no e-mail,
   pois o meu original havia sido clonado ou sei lá o quê! hehe..

Bjos

  - Original Message - 
  From: Marcelo Salhab Brogliato 
  To: obm-l@mat.puc-rio.br 
  Sent: Sunday, October 28, 2007 10:25 PM
  Subject: Re: [obm-l] Questões da OBM


  Olá Shine,
  gostei mto da sua resposta...
  mas nao entendi como vc provou que 1,2,3,4 vai aparecer novamente...

  abracos,
  Salhab



  On 10/27/07, Carlos Yuzo Shine [EMAIL PROTECTED] wrote:
Oi,

O problema 3 tem uma solução bem bonita (não é minha,
eu vi não me lembro onde): imagine que há 100*101/2 =
5050 cordas, cada uma amarrando cada par de pedras.
Então, para Esmeralda separar uma pilha de a+b pedras 
em uma pilha de a pedras e outra de b pedras, ela deve
cortar ab cordas! Como no final devemos ter pedras
soltas, devemos cortar todas as cordas, de modo que a
soma pedida é igual à quantidade de cordas, que é 
5050.

No problema 2, item a, suponha por absurdo que
apareçam 2,0,0,4 nessa ordem. Então, voltando a
seqüência obtemos 2,2,0,0,4; 6,2,2,0,0,4... e só
obtemos números pares, absurdo, pois começamos com
1,2,3,4.

O item b é mais interessante: a seqüência é periódica
(assim como qualquer recursão linear homogênea). Para
ver isso, use casa dos pombos: considere todas as 10^4
quádruplas (a,b,c,d) de algarismos. Agora pense nas 
quádruplas (x,y,z,w) de quatro termos consecutivos da
seqüência dada. Após pelo menos 10^4 + 1 termos,
alguma quádrupla (x,y,z,w) vai se repetir, e a
seqüência vai ciclar a partir daí.

Infelizmente, (x,y,z,w) não é necessariamente 
(1,2,3,4). O que fazer então? Considere o começo da
seqüência mais uma quantidade grande de ciclos (o
suficiente para que seja o dobro do tamanho do começo
da seqüência sem ciclos). Se você voltar a seqüência 
(assim como no item a) de dois pontos diferentes, o
fim do primeiro ciclo e o fim do pedaço considerado da
seqüência, vai obter os mesmos dígitos. Entre eles,
vai aparecer 1,2,3,4 no começo se voltar do primeiro 
ponto e a mesma coisa, 1,2,3,4, se voltar do segundo
ponto. Assim, 1,2,3,4 aparece de novo na seqüência.

[]'s
Shine

--- Marcelo Salhab Brogliato [EMAIL PROTECTED] 
wrote:

 Olá Barola,

 ainda estou tentando resolver.. mas não consegui...
 achei a questão MUITO interessante...
 e espero que o item B seja falso.. é um indicio de
 que a sequencia nao eh 
 periodica..
 resta sabermos se ela nao fica periodica apos um
 tempo... por exemplo:
 aparecendo um segundo 9, 4, 8, 7.. entende?
 entao, poderiamos utiliza-la, por exemplo, para a
 geracao de numeros
 aleatorios...
 uma outra questao interessante é: qual a
 distribuicao de probabilidades
 dessa sequencia?
 como a sequencia esta limitada entre 0 e 9, se
 contarmos qtos 0 
 aparecerem... dps qtos 1 aparecem.. e
 assim por diante... e fizermos n-inf, essas
 quantidades seriam iguais?!

 estou tentando.. se eu conseguir mando alguma
 coisa..
 mas estou realmente sem ideias... 

 junto contigo, fico no aguardo da solucao de alguem
 da lista!

 abraços,
 Salhab




 On 10/24/07, [EMAIL PROTECTED] 
 [EMAIL PROTECTED] wrote:
 
   Oi gente! Alguém pode resolver estas? São da 3ª
 fase da OBM, mas pelo 
  visto o site não disponibiliza o gabarito.
 
 
 
  *PROBLEMA 2*
 
  A seqüência de algarismos
 
  1, 2, 3, 4, 0, 9, 6, 9, 4, 8, 7, … 
 
 
 
  é construída da seguinte maneira: cada elemento, a
 partir do quinto, é
  igual ao último algarismo da soma dos quatro
 anteriores.
  
  a) Os algarismos 2, 0, 0, 4, juntos e nesta ordem,
 aparecem na seqüência?
 
  b) Os algarismos iniciais 1, 2, 3, 4, juntos e
 nesta ordem, aparecem
  novamente na seqüência? 
 
 
 
 
 
  *PROBLEMA 3*
 
  Esmeralda tem uma pilha com 100 pedras. Ela divide
 essa pilha em duas novas pilhas e em seguida 
 multiplica as
 
  quantidades de pedras nessas duas novas pilhas e
 escreve o produto em um quadro. Ela então escolhe
 uma pilha
 
  com mais de uma pedra e repete esse procedimento: 
 a pilha é dividida em duas, as quantidades de pedras
 nessas
 
  duas pilhas são multiplicadas e o produto escrito
 no quadro. Esta operação é realizada até se obter
 apenas pilhas 
 
  com 1 pedra cada. Quais são os possíveis valores
 da soma de todos os produtos escritos no quadro?
 
 
 

[obm-l] Re: [obm-l] Questões da OBM

2007-10-29 Por tôpico barola
Desculpe a ignorância, mas porque não podemos pensar que o ciclo seja com 
um período parcial?
Assim: a,b,c,d,e,f,g,h,i,j,k,l,i,j,k,l,i,j,k,l,i,j,k,l,.
Acho que sua solução está certa, só faltou provar que não dá certo para 
esse caso, concorda?

Mesmo assim, você, o Nicolau e todos os grandes alunos e mestres desta 
lista tem me ensinado muito!
Obrigada mesmo!

- Original Message - 
  From: Fetofs Ashu 
  To: obm-l@mat.puc-rio.br 
  Sent: Monday, October 29, 2007 11:28 AM
  Subject: Re: [obm-l] Questões da OBM


  Salhab e Bárbara,

  1) Vamos andar para trás. Se você tem um grupo (x, y, z, w), só há um termo 
que pode vir antes desses quatro termos, quaisquer sejam eles.
  2) Continuando o processo de 1) temos que todo grupo só pode ser obtido 
através de uma sequência definida.
  3) Um grupo deve se repetir, pois o número de grupos possíveis é finito.
  4) Como na primeira vez que esse grupo apareceu (1, 2, 3, 4) fazia parte da 
sequência, deve fazer na segunda vez também, já que a sequência é única.

  Fernando Oliveira 

Re: [obm-l] Re: [obm-l] Questões da OBM

2007-10-29 Por tôpico Fetofs Ashu
Bárbara,

Lembra do meu ponto 1? Se você tem um grupo (x, y, z, w), só há um termo
que pode vir antes desses quatro termos, quaisquer sejam eles.

Provar o ponto 1 é trivial, já que precisamos de um valor congruente a
w-(z+y+x) mod 10 e não há dois termos dos possíveis membros da sequência (0
a 9) que têm o mesmo valor mod 10.

Mesmo se você não entender de aritmética modular, o ponto 1 é muito
intuitivo. Pegue alguns grupos (x, y, z, w) quaisquer e veja se você
consegue achar dois termos que podem vir antes desses. Você vai logo se
cansar, já que não tem jeito :)

(não continue até entender o que eu disse até agora)

Logo a sequência (i, j, k, l) não pode vir depois de um h e depois de um
l ao mesmo tempo (claro, se considerarmos que h é diferente de l).
Concluímos que a situação proposta é impossível.

Note que para rejeitar um ciclo de período indefinido, precisamos do ponto
2. Como cada grupo (c, d, e, f) só tem um termo que pode antecedê-lo
(chamaremos de b), o grupo (b, c, d, e) também só tem um termo que pode
antecedê-lo (um termo a qualquer). Logo, cada termo só pode vir de uma
sequência definida (por exemplo, os números 1, 1, 1, 3 só podem vir depois
de 0, 8, 9, 2, 9, 0, etc.)

Fernando Oliveira



On 10/29/07, [EMAIL PROTECTED] [EMAIL PROTECTED] wrote:

  Desculpe a ignorância, mas porque não podemos pensar que o ciclo seja
 com um período parcial?
 Assim: a,b,c,d,e,f,g,h,i,j,k,l,i,j,k,l,i,j,k,l,i,j,k,l,.
 Acho que sua solução está certa, só faltou provar que não dá certo
 para esse caso, concorda?

 Mesmo assim, você, o Nicolau e todos os grandes alunos e mestres desta
 lista tem me ensinado muito!
 Obrigada mesmo!

 - Original Message -

 *From:* Fetofs Ashu [EMAIL PROTECTED]
 *To:* obm-l@mat.puc-rio.br
 *Sent:* Monday, October 29, 2007 11:28 AM
 *Subject:* Re: [obm-l] Questões da OBM

 Salhab e Bárbara,

 1) Vamos andar para trás. Se você tem um grupo (x, y, z, w), só há um
 termo que pode vir antes desses quatro termos, quaisquer sejam eles.
 2) Continuando o processo de 1) temos que todo grupo só pode ser obtido
 através de uma sequência definida.
 3) Um grupo deve se repetir, pois o número de grupos possíveis é finito.
 4) Como na primeira vez que esse grupo apareceu (1, 2, 3, 4) fazia parte
 da sequência, deve fazer na segunda vez também, já que a sequência é única.

 Fernando Oliveira




[obm-l] Re: [obm-l] Questões da OBM

2007-10-27 Por tôpico barola
Valeu pela dica!
  - Original Message - 
  From: Fetofs Ashu 
  To: obm-l@mat.puc-rio.br 
  Sent: Friday, October 26, 2007 10:49 PM
  Subject: Re: [obm-l] Questões da OBM


  Seria uma boa idéia procurar nas revistas Eureka, se os problemas são 
relativamente recentes, pois lá é onde o gabarito da 3ª fase é normalmente 
disponibilizado.


[obm-l] Re: [obm-l] QUESTÕES DE CONCURSOS!

2007-01-04 Por tôpico Marcelo Salhab Brogliato

Olá,

no jogo de par ou impar ambos tem a mesma probabilidade de vencer pois:
0+0=0
0+1=1
0+2=2
0+3=3
0+4=4
0+5=5
1+0=1
1+1=2
1+2=3
1+3=4
1+4=5
1+5=6
:
:
continuando, obteremos as sequencias:
0 1 2 3 4 5
1 2 3 4 5 6
2 3 4 5 6 7
3 4 5 6 7 8
4 5 6 7 8 9
5 6 7 8 9 10

contando os pares e os impares, temos a mesma quantidade!

abracos,
Salhab


- Original Message - 
From: Jorge Luis Rodrigues e Silva Luis [EMAIL PROTECTED]

To: obm-l@mat.puc-rio.br
Sent: Thursday, January 04, 2007 10:26 AM
Subject: [obm-l] QUESTÕES DE CONCURSOS!


C sabe que A e B estão viajando pela Europa. Com as informações que dispõe, 
ele estima corretamente que a probabilidade de A estar hoje em Paris é 3/7, 
que a probabilidade de B estar hoje em Paris é 2/7, e que a probabilidade 
de ambos, estarem hoje em Paris é 1/7. C, então, recebe um telefonema de A 
informando que ela está hoje em Paris. Com a informação recebida pelo 
telefonema de A, C agora estima corretamente que a probabilidade de B 
também estar hoje em Paris é igual a: 
(MPU - 04)


Maria ganhou de João nove pulseiras, quatro delas de prata e cinco delas 
de ouro. Maria ganhou de Pedro onze pulseiras, oito delas de prata e tres 
delas de ouro. Maria guarda todas essas pulseiras - e apenas essas - em 
sua pequena caixa de jóias. Uma noite, arrumando-se apressadamente para ir 
ao cinema com João, Maria retira, ao acaso, uma pulseira de sua pequena 
caixa de jóias. Ela vê, então, que retirou uma pulseira de prata. 
Levando-se em conta tais informações, a probabilidade de que a pulseira de 
prata que Maria retirou seja uma das pulseiras que ganhou de João é igual 
a: (MPU - 04)


Um baralho comum contém 52 cartas de 4 tipos (naipes) diferentes: em cada 
naipe, que consiste de 13 cartas, 3 dessas cartas contêm as figuras do 
rei, da dama e do valete, respectivamente. A probabilidade de se extrair 
uma carta e ela conter uma figura ou ser uma carta de paus é igual a:


(TCU - 04)

A propósito, porque no velho jogo de par-ou-ímpar os dois jogadores terão 
sempre a mesma probabilidade de vencer, se aparentemente um deles já 
começa em desvantagem? (BACEN-94)



Divirtam-se!

_
Experimente o novo Windows Live Messenger! 
http://get.live.com/messenger/overview


=
Instruções para entrar na lista, sair da lista e usar a lista em
http://www.mat.puc-rio.br/~nicolau/olimp/obm-l.html
=


--
No virus found in this incoming message.
Checked by AVG Free Edition.
Version: 7.1.409 / Virus Database: 268.16.4/615 - Release Date: 3/1/2007




=
Instruções para entrar na lista, sair da lista e usar a lista em
http://www.mat.puc-rio.br/~nicolau/olimp/obm-l.html
=


[obm-l] Re: [obm-l] QUESTÕES DE CONCURSOS!

2007-01-04 Por tôpico Marcelo Salhab Brogliato

Olá,

A: pulseira de prata
B: pulseira de ouro
C: ganhou de Joao
D: ganhou de Pedro

P(A/C) = P(A inter C)/P(C)
P(C/A) = P(A inter C)/P(A)

= P(A/C) * P(C) = P(C/A) * P(A)

P(A/C) = 4/9 [probabilidade da pulseira ser de prata, dado que ela foi dada 
por Joao]

P(C) = 9/20 [probabilidade da pulseira ser de Joao]
P(A) = 12/20 [probabilidade da pulseira ser de prata]

= P(C/A) = 4/9 * 9/20 * 20/12 = 4/12 = 1/3

abracos,
Salhab

- Original Message - 
From: Jorge Luis Rodrigues e Silva Luis [EMAIL PROTECTED]

To: obm-l@mat.puc-rio.br
Sent: Thursday, January 04, 2007 10:26 AM
Subject: [obm-l] QUESTÕES DE CONCURSOS!


C sabe que A e B estão viajando pela Europa. Com as informações que dispõe, 
ele estima corretamente que a probabilidade de A estar hoje em Paris é 3/7, 
que a probabilidade de B estar hoje em Paris é 2/7, e que a probabilidade 
de ambos, estarem hoje em Paris é 1/7. C, então, recebe um telefonema de A 
informando que ela está hoje em Paris. Com a informação recebida pelo 
telefonema de A, C agora estima corretamente que a probabilidade de B 
também estar hoje em Paris é igual a: 
(MPU - 04)


Maria ganhou de João nove pulseiras, quatro delas de prata e cinco delas 
de ouro. Maria ganhou de Pedro onze pulseiras, oito delas de prata e tres 
delas de ouro. Maria guarda todas essas pulseiras - e apenas essas - em 
sua pequena caixa de jóias. Uma noite, arrumando-se apressadamente para ir 
ao cinema com João, Maria retira, ao acaso, uma pulseira de sua pequena 
caixa de jóias. Ela vê, então, que retirou uma pulseira de prata. 
Levando-se em conta tais informações, a probabilidade de que a pulseira de 
prata que Maria retirou seja uma das pulseiras que ganhou de João é igual 
a: (MPU - 04)


Um baralho comum contém 52 cartas de 4 tipos (naipes) diferentes: em cada 
naipe, que consiste de 13 cartas, 3 dessas cartas contêm as figuras do 
rei, da dama e do valete, respectivamente. A probabilidade de se extrair 
uma carta e ela conter uma figura ou ser uma carta de paus é igual a:


(TCU - 04)

A propósito, porque no velho jogo de par-ou-ímpar os dois jogadores terão 
sempre a mesma probabilidade de vencer, se aparentemente um deles já 
começa em desvantagem? (BACEN-94)



Divirtam-se!

_
Experimente o novo Windows Live Messenger! 
http://get.live.com/messenger/overview


=
Instruções para entrar na lista, sair da lista e usar a lista em
http://www.mat.puc-rio.br/~nicolau/olimp/obm-l.html
=


--
No virus found in this incoming message.
Checked by AVG Free Edition.
Version: 7.1.409 / Virus Database: 268.16.4/615 - Release Date: 3/1/2007




=
Instruções para entrar na lista, sair da lista e usar a lista em
http://www.mat.puc-rio.br/~nicolau/olimp/obm-l.html
=


[obm-l] Re: [obm-l] QUESTÕES DE CONCURSOS!

2007-01-04 Por tôpico Marcelo Salhab Brogliato

Olá,

P(AUB) = P(A) + P(B) - P(AinterB)

A = figura
B = carta de paus

no baralho temos 12 cartas com figura, entao: P(A) = 12/52
no baralho temos 13 cartas de paus, entao: P(B) = 13/52
no baralho temos 3 cartas com figura de paus, entao: P(A inter B) = 3/52

= P(AUB) = 12/52 + 13/52 - 3/52 = 22/52 = 11/26

abraços,
Salhab


- Original Message - 
From: Jorge Luis Rodrigues e Silva Luis [EMAIL PROTECTED]

To: obm-l@mat.puc-rio.br
Sent: Thursday, January 04, 2007 10:26 AM
Subject: [obm-l] QUESTÕES DE CONCURSOS!


C sabe que A e B estão viajando pela Europa. Com as informações que dispõe, 
ele estima corretamente que a probabilidade de A estar hoje em Paris é 3/7, 
que a probabilidade de B estar hoje em Paris é 2/7, e que a probabilidade 
de ambos, estarem hoje em Paris é 1/7. C, então, recebe um telefonema de A 
informando que ela está hoje em Paris. Com a informação recebida pelo 
telefonema de A, C agora estima corretamente que a probabilidade de B 
também estar hoje em Paris é igual a: 
(MPU - 04)


Maria ganhou de João nove pulseiras, quatro delas de prata e cinco delas 
de ouro. Maria ganhou de Pedro onze pulseiras, oito delas de prata e tres 
delas de ouro. Maria guarda todas essas pulseiras - e apenas essas - em 
sua pequena caixa de jóias. Uma noite, arrumando-se apressadamente para ir 
ao cinema com João, Maria retira, ao acaso, uma pulseira de sua pequena 
caixa de jóias. Ela vê, então, que retirou uma pulseira de prata. 
Levando-se em conta tais informações, a probabilidade de que a pulseira de 
prata que Maria retirou seja uma das pulseiras que ganhou de João é igual 
a: (MPU - 04)


Um baralho comum contém 52 cartas de 4 tipos (naipes) diferentes: em cada 
naipe, que consiste de 13 cartas, 3 dessas cartas contêm as figuras do 
rei, da dama e do valete, respectivamente. A probabilidade de se extrair 
uma carta e ela conter uma figura ou ser uma carta de paus é igual a:


(TCU - 04)

A propósito, porque no velho jogo de par-ou-ímpar os dois jogadores terão 
sempre a mesma probabilidade de vencer, se aparentemente um deles já 
começa em desvantagem? (BACEN-94)



Divirtam-se!

_
Experimente o novo Windows Live Messenger! 
http://get.live.com/messenger/overview


=
Instruções para entrar na lista, sair da lista e usar a lista em
http://www.mat.puc-rio.br/~nicolau/olimp/obm-l.html
=


--
No virus found in this incoming message.
Checked by AVG Free Edition.
Version: 7.1.409 / Virus Database: 268.16.4/615 - Release Date: 3/1/2007




=
Instruções para entrar na lista, sair da lista e usar a lista em
http://www.mat.puc-rio.br/~nicolau/olimp/obm-l.html
=


[obm-l] Re: [obm-l] QUESTÕES DE CONCURSOS!

2007-01-04 Por tôpico Marcelo Salhab Brogliato

Olá,

P(A) = 3/7
P(B) = 2/7
P(A inter B) = 1/7

P(B|A) = P(AinterB)/P(A) = 1/7 * 7/3 = 1/3

abraços,
Salhab


- Original Message - 
From: Jorge Luis Rodrigues e Silva Luis [EMAIL PROTECTED]

To: obm-l@mat.puc-rio.br
Sent: Thursday, January 04, 2007 10:26 AM
Subject: [obm-l] QUESTÕES DE CONCURSOS!


C sabe que A e B estão viajando pela Europa. Com as informações que dispõe, 
ele estima corretamente que a probabilidade de A estar hoje em Paris é 3/7, 
que a probabilidade de B estar hoje em Paris é 2/7, e que a probabilidade 
de ambos, estarem hoje em Paris é 1/7. C, então, recebe um telefonema de A 
informando que ela está hoje em Paris. Com a informação recebida pelo 
telefonema de A, C agora estima corretamente que a probabilidade de B 
também estar hoje em Paris é igual a: 
(MPU - 04)


Maria ganhou de João nove pulseiras, quatro delas de prata e cinco delas 
de ouro. Maria ganhou de Pedro onze pulseiras, oito delas de prata e tres 
delas de ouro. Maria guarda todas essas pulseiras - e apenas essas - em 
sua pequena caixa de jóias. Uma noite, arrumando-se apressadamente para ir 
ao cinema com João, Maria retira, ao acaso, uma pulseira de sua pequena 
caixa de jóias. Ela vê, então, que retirou uma pulseira de prata. 
Levando-se em conta tais informações, a probabilidade de que a pulseira de 
prata que Maria retirou seja uma das pulseiras que ganhou de João é igual 
a: (MPU - 04)


Um baralho comum contém 52 cartas de 4 tipos (naipes) diferentes: em cada 
naipe, que consiste de 13 cartas, 3 dessas cartas contêm as figuras do 
rei, da dama e do valete, respectivamente. A probabilidade de se extrair 
uma carta e ela conter uma figura ou ser uma carta de paus é igual a:


(TCU - 04)

A propósito, porque no velho jogo de par-ou-ímpar os dois jogadores terão 
sempre a mesma probabilidade de vencer, se aparentemente um deles já 
começa em desvantagem? (BACEN-94)



Divirtam-se!

_
Experimente o novo Windows Live Messenger! 
http://get.live.com/messenger/overview


=
Instruções para entrar na lista, sair da lista e usar a lista em
http://www.mat.puc-rio.br/~nicolau/olimp/obm-l.html
=


--
No virus found in this incoming message.
Checked by AVG Free Edition.
Version: 7.1.409 / Virus Database: 268.16.4/615 - Release Date: 3/1/2007




=
Instruções para entrar na lista, sair da lista e usar a lista em
http://www.mat.puc-rio.br/~nicolau/olimp/obm-l.html
=


[obm-l] Re: [obm-l] Questões do Livro do Hefez

2006-05-02 Por tôpico Artur Costa Steiner
2) Vejamos o caso o caso a^p - a. Temos que o primo p= 5 eh impar, e desta
forma p-1 eh par. Assim, p-1 = 2p' para algum inteiro positivo p'.

Temos que a^p - a = a(a^(p-1) -1) = a(a^(2p') - 1) = a(a^p' + 1)(a^p' - 1).
Se a for par, entao eh imediato que a^p - a eh par. Se a for impar, entao
a^p'eh impar e os numeros (a^p' + 1) e (a^p' - 1) sao ambos pares. Logo,
tambem neste caso a^p - a eh par, sendo inclusive multiplo de 4.

Se a for multiplo de 3, entao eh imediato que a^p - a eh tambem multiplo de
3. Se a nao for multiplo de 3, temos 2 casos: se a for par, a^p' eh par.
Logo, um dos numeros (a^p' + 1) ou (a^p -1) eh multiplo de 3 (para todo
numero par n que nao seja multiplo de 3, n-1 ou n +1 eh multiplo de 3). Isto
nos mostra que a^p - a eh multiplo de 3. Se a nao for multiplo de 3 e for
impar, entao a^p' eh um impar nao multiplo de 3. Entao, dentre os numeros
pares (a^p' + 1) e (a^p' - 1) um deles eh necessariamente multiplo de 3 (se
n eh um impar nao multiplo de 3, entao um dos pares n-1 e n+1 eh sempre
multiplo de 3).
Chegamos assim aa conclusao de que, nas condicoes dadas, a^p - a eh sempre
par e multiplo de 3, logo eh multiplo de 6. 

Pelo pequeno teorema de Fermat, temos ainda que a^p = a (mod p). Logo a^p -
a eh multiplo de p. E como para o primo p temos  p 3, segue-se, em virtude
da conclusao anterior, que a^p - a eh multiplo de 6p, ou seja, 6p divide a^p
- p, conforme afirmado.

O outro deve ter um saida semelhante, depois vemos se dah pra sair.
De uma conferida, meu conhecimento de teoria dos numeros eh muito limitado.
Artur


  Agradeço  qualquer ajuda nas seguintes questões:
 
  1) Mostre que existe uma correspondência biunívoca
  entre pares de primos
  gêmeos e números n tais que n^2 -1 possui 4
  divisores.
 
  2) Seja p 3 um primo. Mostre que a^p - a  e a^p. b-
  b^p . a são divisíveis
  por 6p, para todos a0, com ab.
 
  3) seja p um primo ímpar. Mostre que se pode
  escrever p = y^2 - x^2, com  x
  e y positivos, de modo único.
 
  Obrigado
=

=
Instruções para entrar na lista, sair da lista e usar a lista em
http://www.mat.puc-rio.br/~nicolau/olimp/obm-l.html
=


[obm-l] Re: [obm-l] Questões do Livro do Hefez

2006-05-01 Por tôpico Ricardo Khawge

Desculpe Artur, já encontrei a mensagem



From: Artur Costa Steiner [EMAIL PROTECTED]
Reply-To: obm-l@mat.puc-rio.br
To: obm-l@mat.puc-rio.br
Subject: Re: [obm-l] Questões do Livro do Hefez
Date: Sun, 30 Apr 2006 23:55:37 -0700 (PDT)

1) Suponhamos que m = n^2 - 1 = (n+1)(n-1)  possua 4
divisores. Temos que n=3. Se n for impar, entao n- 1
e n+1 sao ambos pares, implicando que m seja multiplo
de 4. Se n =3, entao m =8 tem 4 divisores, mas isto
nao leva ainda aa correspondencia desejada. Se n=5
for impar, entao os numeros pares n-1 =4 e n+1 sao
divisores de m. Alem disto, m tem como divisores os
numeros 1 , 2 e m, de modo que para n=5, impar, m tem
pelo menos 5 divisores, contraraiamente aa hipotese.
Assim, valore impares de n nao implicam a
correspondencia de4sejada.
Se n=4 for par, entao n-1=3 e n+1 sao ambos
divisores impares de m. Alem disto, m tem por
divisores os numeros 1 e o proprio m. Dado que m tem
exatamente 4 divisores, segue-se que n-1 e n+1 sao
ambos primos, pois, se ao menos um deles fosse
composto, m teria pelo menos um divisor a mais do que
os citados, contrariamente aa hipotese basica.
Concluimos assim que, a cada valor par de n para o
qual n-1 e n+1 sejam primos - logo primos gemeos -
corresponde o par (n-1 , n+1) de primos gemeos.
Por outro lado, se n-1 e n+1 sao pimos gemeos, entao m
= n^2 -1 = (n-1)(n+1) tem por fatores primos unica e
exclusivamente n-1 e n+1 (teorema fundamental da
aritmetica). Como, alem disto, 1 e m sao divisores de
m, segue-se que m tem exatamente 4 divisores. Isto eh,
a cada par de primos gemeos, corresponde um numero da
forma n^2 -1. Concluimos, assim, que a correspondencia
entre o conjunto dos pares de primos gemeos e os
numeros da forma n^2 -1 eh ,de fato, biunivica, hah
uma bijecao entre os 2 conjuntos.

A questao 3 jah foi discutida na lista, de forma mais
geral, hah alguns dias, sob o titulo Diferenca de 2
quadrados. Basta fazer y = (p+1)/2 e x = (p-1)/2.

A questao 2 parece mais complicada, vamos tentar outra
hora.

Artur

--- Ricardo Khawge [EMAIL PROTECTED] wrote:

 Agradeço  qualquer ajuda nas seguintes questões:

 1) Mostre que existe uma correspondência biunívoca
 entre pares de primos
 gêmeos e números n tais que n^2 -1 possui 4
 divisores.

 2) Seja p 3 um primo. Mostre que a^p - a  e a^p. b-
 b^p . a são divisíveis
 por 6p, para todos a0, com ab.

 3) seja p um primo ímpar. Mostre que se pode
 escrever p = y^2 - x^2, com  x
 e y positivos, de modo único.

 Obrigado


_
 Ganhe tempo encontrando o arquivo ou e-mail que você
 precisa com Windows
 Desktop Search. Instale agora em
 http://desktop.msn.com.br


=
 Instruções para entrar na lista, sair da lista e
 usar a lista em
 http://www.mat.puc-rio.br/~nicolau/olimp/obm-l.html

=



__
Do You Yahoo!?
Tired of spam?  Yahoo! Mail has the best spam protection around
http://mail.yahoo.com
=
Instruções para entrar na lista, sair da lista e usar a lista em
http://www.mat.puc-rio.br/~nicolau/olimp/obm-l.html
=


_
Ligações gratuitas de PC-para-PC para qualquer lugar do Brasil e do mundo 
com o  MSN Messenger. Saiba mais em   
http://imagine-msn.com/messenger/default2.aspx?locale=pt-br


=
Instruções para entrar na lista, sair da lista e usar a lista em
http://www.mat.puc-rio.br/~nicolau/olimp/obm-l.html
=


[obm-l] Re:[obm-l] questões olim internac ional

2005-12-21 Por tôpico lponce
Ao amigo da obm
vai uma sugestão para o problema :

(Moldávia – 2000) Os números inteiros a,b,c satisfazem à relação a + b + c = 0. Mostre que o número 2(a4 + b 4 +c4 )é um quadrado perfeito.
Uma possível demonstração . Desde que a + b + c = 0, 

 c = - (a + b)  [ 1 ]
 c2 = a2 + 2ab + b2   [ 2 ]
a2 + b2 + c2 + 2 (ab + c (a+b) ) = 0   [ 3]
Nestas condições:
Elevando ambos membros de [3] ao quadrado e substituindo [ 1] encontramos: 
 a2 + b2 + c2 = 2 (a2 +ab+ b2 )  [ 4]
Elevando ambos membros de [4] ao quadrado e substituindo [ 2] encontramos:
a4 + b 4 + c4 + 2 [a2b2 + (a2 + b2) (a2 + 2ab + b2)]=4[a4 +3a2b2 +2 a3 b+2 ab3 +b4]
Simplificando e agrupando de modo conveniente, obtém-se sucessivamente:
a4 + b 4 + c4 = 2[(a4 + a3b + a2b2 ) + (a3 b + a2b2 + a b3 ) + (a2b2 + a b3 + b4 )]
a4 + b 4 + c4 = 2[a2 (a2 + a b + b2 ) + ab(a2 + a b + b2 ) + b2 (a2 + a b + b2 )]
Dai, 
2a4 +2b 4 +2 c4 = 4[a2 (a2 + a b + b2 ) + ab(a2 + a b + b2 ) + b2 (a2 + a b + b2 )]
 = [ 2 (a2 + a b + b2 ) ]2 ,
Portanto, para a, b e c inteiros tais que: a + b + c = 0, segue-se que
2a4 +2b 4 +2 c4 =[2 (a2 + a b + b2 ) ]2 , isto é, que o número 2(a4 + b 4 +c4 )é um quadrado perfeito, o que finaliza a demonstração.

Do amigo PONCE
Aproveitando este Email ,gostaria a desejar a todos os membros desta lista um feliz natal e ummaravilhoso 2006.






De:
[EMAIL PROTECTED]




Para:
obm-l@mat.puc-rio.br




Cópia:





Data:
Fri, 16 Dec 2005 16:00:12 -0300 (ART)




Assunto:
[obm-l] questões olim internacional
 valeu para aqueles que me ajudaram resolvendo as 2 questões anteriores .
 
 agora quem puder ajuda nessa eu agradeço !
 http://img29.imagevenue.com/img.php?loc=loc298image=64fe3_fagner3.JPG


Yahoo! doce lar. Faça do Yahoo! sua homepage. 


[]a, L.PONCE.


[obm-l] Re:[obm-l] questões!!

2005-11-14 Por tôpico Luiz H\. Barbosa

Quando eu estudava pro colégio naval ,usava :Problemas selecionados de algebra e analise (Jorge W. e Morgado)
Eu acho q é isso , o livro tem uma capinha amarelinha.
Infelizmente não sei onde o meu foi parar.






-- Início da mensagem original --- 

De: [EMAIL PROTECTED] 
Para: obm-l@mat.puc-rio.br 
Cc: 
Data: Mon, 14 Nov 2005 00:18:18 -0200 
Assunto: [obm-l] questões!! 

alguem sabe onde eu posso encontrar questões boas, ótimas de 5º a 8º série ?? 
e também livros ?? 

=== 


[obm-l] Re:[obm-l] questões!!

2005-11-14 Por tôpico Paulo Santa Rita

Ola Marcio,

Voce pode achar aqui :

http://www.mat.puc-rio.br/~nicolau/psr

Um abraco
Paulo Santa Rita
2,1425,141105


From: marcio aparecido [EMAIL PROTECTED]
Reply-To: obm-l@mat.puc-rio.br
To: obm-l@mat.puc-rio.br
Subject: Re: [obm-l] Re:[Spam] [obm-l] questões!!
Date: Mon, 14 Nov 2005 11:11:37 -0200

questões para praticar, pegar velocidade na resoluções.
pode ter questões de todos os níveis (fácil, media e dificil).


_
Chegou o que faltava: MSN Acesso Grátis. Instale Já! 
http://www.msn.com.br/discador


=
Instruções para entrar na lista, sair da lista e usar a lista em
http://www.mat.puc-rio.br/~nicolau/olimp/obm-l.html
=


Re: [obm-l] Re:[obm-l] questões!!

2005-11-14 Por tôpico marcio aparecido
Obrigado pelas questões Paulo.
Alguem pode me ajudar com livros também.
Abraços Márcio.

=
Instruções para entrar na lista, sair da lista e usar a lista em
http://www.mat.puc-rio.br/~nicolau/olimp/obm-l.html
=


[obm-l] RE: [obm-l] Questões complicadas

2005-04-23 Por tôpico Rafael Alfinito Ferreira
QUESTÃO 1
Para z ser mínimo quanto mais fatores zero mehor, logo:
3x+4y=12 -- x=4.(3-y)/3;se y=0, x=4
y=3(4-x)/4;se x=0, y=3
Logicamente o segundo par ordenado (x,y)=(0,3) é o mais indicado.
sendo assim z = 9
A QUESTÃO 2 EU NÃO CONSEGUI COMPREENDER, TEM CERTEZA QUE O ENUNCIADO ESTÁ 
CORRETO

UM GRANDE ABRAÇO!
RAFAEL!
From: Robÿe9rio Alves [EMAIL PROTECTED]
Reply-To: obm-l@mat.puc-rio.br
To: obm-l@mat.puc-rio.br
Subject: [obm-l] Questões complicadas
Date: Sat, 23 Apr 2005 19:22:20 -0300 (ART)
01) Se x e y são reais tais que 3x + 4y = 12, determinar o valor mínimo de 
z = x^2 + y^2 .

02)  Um avião de 100 lugares foi fretado para uma excursão. A companhia de 
cada passageiro R$ 800,00 mais R$ 10,00 por cada lugar vago. Para que 
número de passageiro a rentabilidade da empresa é máxima ?


-
Yahoo! Acesso Grátis: Internet rápida e grátis. Instale o discador agora!
_
MSN Messenger: converse online com seus amigos .  
http://messenger.msn.com.br

=
Instruções para entrar na lista, sair da lista e usar a lista em
http://www.mat.puc-rio.br/~nicolau/olimp/obm-l.html
=


[obm-l] Re: [obm-l] Questões estranhas

2004-08-04 Por tôpico willian kanashiro

uma dica para a questao 1, use aquele teorema que diz:
"Se ouver raizes racionais elas serao parte do conjunto divisores do termo independente pelos divisores do primeiro termo dependente"
eu resolvi esta questao na prova a partir desse teorema
se tiver problemas me mande um e-mail que eu faço ela comentada 

From: victor machado [EMAIL PROTECTED] 
Reply-To: [EMAIL PROTECTED] 
To: [EMAIL PROTECTED] 
Subject: Re: [obm-l] Questões estranhas 
Date: Tue, 3 Aug 2004 20:02:35 -0300 
 
... "para qualquer valor de 'x' nesta equacao, ' X ' sera um multiplo de 10 "... 
 
On Tue, 3 Aug 2004 18:12:25 -0300, victor machado [EMAIL PROTECTED] wrote: 
  2 - pensa assim : 
  pra qualquer valor de de "x" nesta equacao, sempre vai ser um multiplo 
  de 10, assim: 
  12.10.x^4 + 10.x^2 + 8, pra qualquer numero que voce colocar no "x" , 
  o numero formado será terminado em "8", logo nao existe nenhum valor 
  para x que faca com que este numero seja quadrado perfeito, pois nao 
  ha quadrado perfeito terminado em 8 
  
  On Mon, 2 Aug 2004 21:21:20 -0300, Henrique Patrício Sant'Anna Branco 
  
  
  [EMAIL PROTECTED] wrote: 
   Alguém poderia me dar uma ajuda nisso? 
   
   1 - Sabendo-se que a equação x^2*(x + 13) - 6x*(x^2 + 2) + 4 = 0 pode ser 
   escrita como o produto de dois binômios do primeiro grau, a soma de duas das 
   suas raízes distintas é igual a: 
   Resp.: 3 
   
   2 - O valor numérico da expressão 120x^4 + 10k^2 + 8, sendo k um natural, é 
   o quadrado de um número natural para: 
   Resp.: Nenhum valor de k 
   
   Esse eu assumi que a equação pudesse ser fatorada como ((x - a)(x - b))^2 ou 
   (x - c)^4 para ser um quadrado perfeito, resolvi a biquadrada e aí se chega 
   à conclusão que não existe nenhuma raiz natural (nem mesmo real) dessa 
   equação. É o modo certo de fazer? 
   
   Grato, 
   Henrique. 
   
   = 
   Instruções para entrar na lista, sair da lista e usar a lista em 
   http://www.mat.puc-rio.br/~nicolau/olimp/obm-l.html 
   = 
   
  
 
= 
Instruções para entrar na lista, sair da lista e usar a lista em 
http://www.mat.puc-rio.br/~nicolau/olimp/obm-l.html 
= 
MSN Messenger: converse com os seus amigos online.  Instale grátis. Clique aqui.  
=
Instruções para entrar na lista, sair da lista e usar a lista em
http://www.mat.puc-rio.br/~nicolau/olimp/obm-l.html
=


[obm-l] RE: [obm-l] Re: [obm-l] Re: [obm-l] Questões estranhas

2004-08-04 Por tôpico willian kanashiro

discordo sobre a questao da estranhesa da questao sobre medias
essa questao e mais do que possivel e pede um grau de racionio logico 
pense comigo:
uma equaçao do segundo grau pode ser escrito da seguinte forma
ax^2+bx+c=0
assim a media aritmetica é "b:2a' a geometrica é "raiz(c:a)"e a armonica é "2c:b"
From: "Paulo Rodrigues" [EMAIL PROTECTED] 
Reply-To: [EMAIL PROTECTED] 
To: [EMAIL PROTECTED] 
Subject: [obm-l] Re: [obm-l] Re: [obm-l] Questões estranhas 
Date: Tue, 3 Aug 2004 17:04:49 -0300 
 
Acho que não fui claro... 
 
Na questão 1 da prova está escrito: 
 
"Qual é o produto notável representado, geometricamente, na figura acima, na 
qual ABCD é um retângulo? 
 
A figura mosta um retângulo e tenta induzir a expansão de 
(a+b)^2=a^2+2ab+b^2. 
 
Mas não existe produto notável representado na questão! Na figura não se faz 
nem menção a área... 
 
Posso estar sendo demais formal, porém, essa questão é imbecil para quem 
conhece e indecifrável para quem não conhece. 
 
Quanto a questão 16 você tem razão...interpretei quociente no lugar de 
resto. 
 
Em compensação a prova tem várias questões estranhas, que não levam a nada, 
como a 4, que pergunta se é possível calcular a média aritmética, a média 
geométrica e a harmônica, de dois números positivos. 
 
 
- Original Message - 
From: "Marcos Paulo" [EMAIL PROTECTED] 
To: [EMAIL PROTECTED] 
Sent: Tuesday, August 03, 2004 4:16 PM 
Subject: Re: [obm-l] Re: [obm-l] Questões estranhas 
 
 
Oi Paulo, 
eu discordo da estranheza da prova. Achei que a prova foi bastante 
interessante ressucitando temas interessantes que estavam meio que às 
traças como o retângulo áureo (questão 7); o eixo radical (questão 17), a 
fórmula de transformação de radicais duplos em soma de radicais simples 
(questão 2). O produto notável pedido na questão 1 aparece em quase todo 
livro de oitava série (mesmo os piores) e eu não conheço outra 
justificativa (ou uma melhor) para que uma divisão entre inteiros resulte 
numa dízima periódica a não ser o fato de que exista uma quantidfade finita 
de restos possíveis na divisão, enquanto o processo (o algoritmo da 
divisão) pode ser repetido infinitamente. 
Talvez a opinião dos outros membros da lista fosse interessante nessa 
questão. 
 
[]'s MP 
 
P.S. Os números das questões que eu citei são referentes à prova azul. 
 
At 14:12 3/8/2004, you wrote: 
 
 Essa prova do CN está esquisita mesmo. Vocês viram as questões 1 e 16? 
  
 No caso da 16, a resposta certa é a única que faz algum sentido, mas dá a 
 entender que toda seq. com uma quantidade limitada de valores é periódica! 
  
 Essas provas do CN já não foram melhores? 
  
 Paulo 
 
 
-- 
Mensagens enviadas estão livres de vírus. 
Verificado por AVG Anti-Vírus (http://www.avgbrasil.com.br). 
Versão: 7.0.262 / Banco de dados de Vírus: 264.2.0 - Data de Lançamento: 
2/8/2004 
 
 
 
= 
Instruções para entrar na lista, sair da lista e usar a lista em 
http://www.mat.puc-rio.br/~nicolau/olimp/obm-l.html 
= 
 
 
--- 
Outgoing mail is certified Virus Free. 
Checked by AVG anti-virus system (http://www.grisoft.com). 
Version: 6.0.733 / Virus Database: 487 - Release Date: 2/8/2004 
 
= 
Instruções para entrar na lista, sair da lista e usar a lista em 
http://www.mat.puc-rio.br/~nicolau/olimp/obm-l.html 
= 
MSN Messenger: converse com os seus amigos online.  Instale grátis. Clique aqui.  
=
Instruções para entrar na lista, sair da lista e usar a lista em
http://www.mat.puc-rio.br/~nicolau/olimp/obm-l.html
=


Re: [obm-l] Re: [obm-l] Questões estranhas

2004-08-04 Por tôpico Paulo-Andre . Melo




Estava estranhando que ninguem falava dessa questao. Na hora que fizemos
ficou claro que a redacao fora um pouco infeliz. Como jah vi a banca do
colegio manter opcoes piores que essa como corretas acho que ainda vai dar
pano para manga. A questao seria de portugues e estaria centrada na palavra
DISTINTOS, ou seja, plural. Dessa depreende-se que a quesao continha uma
armadilha interpretativa sutil (acreditam os elaboradores da questao).
Sendo entao unica possibilidade correta (sob esse ponto de vista) a
resposta 2.

No mais acho que sempre existirao questoes faceis (de pouco valor agregado)
medias e inteligentes.

Coisas do Naval ...

Paulo Andr B. Melo



   
 
  Augusto Cesar   
 
  de Oliveira  To:  [EMAIL PROTECTED]  
  
  Morgado cc: 
 
  [EMAIL PROTECTED] Subject: Re: [obm-l] Re: [obm-l] 
Questes estranhas  
  n.com.br
 
  Sent by: 
 
  [EMAIL PROTECTED]
  
  puc-rio.br   
 
   
 
   
 
  03/08/2004 20:13 
 
  Please respond   
 
  to obm-l 
 
   
 
   
 




Na questao 18, o numero procurado pode ser 1 ou 2.

==
Mensagem  enviada  pelo  CIP  WebMAIL  - Nova Gerao - v. 2.1
CentroIn Internet Provider  http://www.centroin.com.br
Tel: (21) 2542-4849, (21) 2295-3331Fax: (21) 2295-2978
Empresa 100% Brasileira - Desde 1992 prestando servicos online


-- Original Message ---
From: Marcos Paulo [EMAIL PROTECTED]
To: [EMAIL PROTECTED]
Sent: Tue, 03 Aug 2004 16:16:31 -0300
Subject: Re: [obm-l] Re: [obm-l] Questes estranhas

 Oi Paulo,
 eu discordo da estranheza da prova. Achei que a prova foi bastante
 interessante ressucitando temas interessantes que estavam meio que
 s traas como o retngulo ureo (questo 7); o eixo radical
 (questo 17), a frmula de transformao de radicais duplos em soma
 de radicais simples
 (questo 2). O produto notvel pedido na questo 1 aparece em quase
 todo livro de oitava srie (mesmo os piores) e eu no conheo outra
 justificativa (ou uma melhor) para que uma diviso entre inteiros
 resulte numa dzima peridica a no ser o fato de que exista uma
 quantidfade finita de restos possveis na diviso, enquanto o
 processo (o algoritmo da diviso) pode ser repetido infinitamente.
 Talvez a opinio dos outros membros da lista fosse interessante
 nessa questo.

 []'s MP

 P.S. Os nmeros das questes que eu citei so referentes  prova azul.

 At 14:12 3/8/2004, you wrote:

 Essa prova do CN est esquisita mesmo. Vocs viram as questes 1 e 16?
 
 No caso da 16, a resposta certa  a nica que faz algum sentido, mas d
a
 entender que toda seq. com uma quantidade limitada de valores 
peridica!
 
 Essas provas do CN j no foram melhores?
 
 Paulo

 --
 Mensagens enviadas esto livres de vrus.
 Verificado por AVG Anti-Vrus (http://www.avgbrasil.com.br).
 Verso: 7.0.262 / Banco de dados de Vrus: 264.2.0  Data de
 Lanamento: 2/8/2004

 =
 Instrues para entrar na lista, sair da lista e usar a lista em
 http://www.mat.puc-rio.br/~nicolau/olimp/obm-l.html
 =
--- End of Original Message ---


[UTF-8?]Re: [obm-l] Re: [obm-l] Questões estranhas

2004-08-04 Por tôpico Augusto Cesar de Oliveira Morgado
Ponto de vista doido esse. Por ele, não se poderia perguntar quantos são os 
numeros inteiros compreendidos entre 3 e 5. Ou quantas sao as raízes 
distintas de x^2-4x+4=0. Falando claro: pisaram na bola!

==
Mensagem  enviada  pelo  CIP  WebMAIL  - Nova Geração - v. 2.1
CentroIn Internet Provider  http://www.centroin.com.br
Tel: (21) 2542-4849, (21) 2295-3331Fax: (21) 2295-2978
Empresa 100% Brasileira - Desde 1992 prestando servicos online


-- Original Message ---
From: [EMAIL PROTECTED]
To: [EMAIL PROTECTED]
Cc: [EMAIL PROTECTED], [EMAIL PROTECTED]
Sent: Wed, 4 Aug 2004 09:28:26 -0300
[UTF-8?]Subject: Re: [obm-l] Re: [obm-l]  Questões estranhas

 Estava estranhando que ninguem falava dessa questao. Na hora que fizemos
 ficou claro que a redacao fora um pouco infeliz. Como jah vi a banca 
 do colegio manter opcoes piores que essa como corretas acho que 
 ainda vai dar pano para manga. A questao seria de portugues e 
 estaria centrada na palavra DISTINTOS, ou seja, plural. Dessa 
 depreende-se que a quesao continha uma armadilha interpretativa 
 sutil (acreditam os elaboradores da questao). Sendo entao unica 
 possibilidade correta (sob esse ponto de vista) a resposta 2.
 
 No mais acho que sempre existirao questoes faceis (de pouco valor 
 agregado) medias e inteligentes.
 
 Coisas do Naval ...
 
 Paulo André B. Melo
 
   Augusto Cesar
 
 de Oliveira  To:  [EMAIL PROTECTED]
 
   Morgado cc:
 
 [EMAIL PROTECTED] Subject: Re: [obm-l] Re: [obm-l] Questões 
 estranhas  
   n.com.br
 
 Sent by:
 
   [EMAIL PROTECTED]   
  puc-
 rio.br
 
   03/08/2004 20:13
 
 Please respond
 
   to obm-l
 
 Na questao 18, o numero procurado pode ser 1 ou 2.
 
 ==
 Mensagem  enviada  pelo  CIP  WebMAIL  - Nova Geração - v. 2.1
 CentroIn Internet Provider  http://www.centroin.com.br
 Tel: (21) 2542-4849, (21) 2295-3331Fax: (21) 2295-2978
 Empresa 100% Brasileira - Desde 1992 prestando servicos online
 
 -- Original Message ---
 From: Marcos Paulo [EMAIL PROTECTED]
 To: [EMAIL PROTECTED]
 Sent: Tue, 03 Aug 2004 16:16:31 -0300
 Subject: Re: [obm-l] Re: [obm-l] Questões estranhas
 
  Oi Paulo,
  eu discordo da estranheza da prova. Achei que a prova foi bastante
  interessante ressucitando temas interessantes que estavam meio que
  às traças como o retângulo áureo (questão 7); o eixo radical
  (questão 17), a fórmula de transformação de radicais duplos em soma
  de radicais simples
  (questão 2). O produto notável pedido na questão 1 aparece em quase
  todo livro de oitava série (mesmo os piores) e eu não conheço outra
  justificativa (ou uma melhor) para que uma divisão entre inteiros
  resulte numa dízima periódica a não ser o fato de que exista uma
  quantidfade finita de restos possíveis na divisão, enquanto o
  processo (o algoritmo da divisão) pode ser repetido infinitamente.
  Talvez a opinião dos outros membros da lista fosse interessante
  nessa questão.
 
  []'s MP
 
  P.S. Os números das questões que eu citei são referentes à prova azul.
 
  At 14:12 3/8/2004, you wrote:
 
  Essa prova do CN está esquisita mesmo. Vocês viram as questões 1 e 16?
  
  No caso da 16, a resposta certa é a única que faz algum sentido, mas dá
 a
  entender que toda seq. com uma quantidade limitada de valores é
 periódica!
  
  Essas provas do CN já não foram melhores?
  
  Paulo
 
  --
  Mensagens enviadas estão livres de vírus.
  Verificado por AVG Anti-Vírus (http://www.avgbrasil.com.br).
[UTF-8?]  Versão: 7.0.262 / Banco de dados de Vírus: 264.2.0 – Data de
  Lançamento: 2/8/2004
 
  =
  Instruções para entrar na lista, sair da lista e usar a lista em
  http://www.mat.puc-rio.br/~nicolau/olimp/obm-l.html
  =
 --- End of Original Message ---
 
 =
 Instruções para entrar na lista, sair da lista e usar a lista em
 http://www.mat.puc-rio.br/~nicolau/olimp/obm-l.html
 =
 
 --
 Le présent message ainsi que ses éventuelles pièces jointes est
 exclusivement destiné au(x) destinataire(s), personnes physiques ou
 morales, qu'il désigne.
 Il constitue de ce fait une correspondance à caractère privé et peut
 contenir des informations confidentielles.
 Si ce message vous est parvenu par erreur, nous vous remercions d'en 
 aviser immédiatement l'expéditeur

[obm-l] Re: [obm-l] Questões estranhas

2004-08-04 Por tôpico fgb1


1 - Sabendo-se que a equação x^2 (x^2 + 13) - 6x(x^2 + 2) + 4 = 0 pode ser
escrita como o produto de dois binômios do primeiro grau, a soma de duas das
suas raízes distintas é igual a:
Resp.: 3

Uma outra maneira de resolver essa questão é a seguinte:

Desenvolvendo as multiplicações tem-se que:

x^4+ 13.x^2 - 6.x^3-12x + 4 = 0

Repare que a soma dos coeficientes dessa equação é igual a 1, logo é divisível pelo binômio (x-1)
Dividindo, econtra-se como quociente a seguinte equação:

(x-1).(x^3 - 5.x^2 + 8.x - 4) = 0

que novamente tem soma dos coeficientes igual a 1, logo pode ser divisível pwlo binômio (x-1)
Dividindo, econtra-se como quociente a seguinte equação:

(x-1).(x-1).(x^2 - 4.x + 4) = 0

que fatorando, teremos:

(x-1)^2.(x-2)^2 = 0

Raízes: +1,+2

Logo a soma é igual a 3
















De:
[EMAIL PROTECTED]




Para:
[EMAIL PROTECTED]




Cópia:





Data:
Tue, 03 Aug 2004 13:38:51 -0300




Assunto:
Re: [obm-l] Questões estranhas






 At 21:21 2/8/2004, you wrote:
 
 Alguém poderia me dar uma ajuda nisso?
 
 1 - Sabendo-se que a equação x^2*(x + 13) - 6x*(x^2 + 2) + 4 = 0 pode ser
 escrita como o produto de dois binômios do primeiro grau, a soma de duas das
 suas raízes distintas é igual a:
 Resp.: 3
 
 Essa questão vc copiou errada a pergunta correta é:
 Sabendo que x^2 (x^2 + 13) - 6x(x^2 + 2) + 4 = 0 pode ser escrita como o 
 produto de binomios do primeiro grau (e não de DOIS binomios)..
 
 Note que o termo independente de x vale 4 e, portanto se houver alguma raiz 
 inteira essa será um dos divisores de 4, ou seja, +1, -1, +2, -2, +4, -4. 
 Basta testar 1 e 2 e vc verá que são raízes.
 
 2 - O valor numérico da expressão 120x^4 + 10k^2 + 8, sendo k um natural, é
 o quadrado de um número natural para:
 Resp.: Nenhum valor de k
 
 
 note que 120k^4 tem digito das unidades ZERO, asssim como 10k^2. POrtanto o 
 dígito das unidades da expressão inteira será 8 e não existe quadrado 
 perfeito que termine em 8, portanto independente do valor inteiro de k, a 
 expressão nunca será um quadrado perfeito.
 
 Quem quiser ver a prova inteira do colégio naval pode entrar no endereço 
 www.cursoriachuelo.com.br/cn2004.htm neste endereço está o gabarito extra 
 oficial e clicando nas questões da prova azul abre uma janelinha com a 
 questão correspondente (enunciado + opções).
 Se tiver um tempinho eu vou colocar em breve a prova com soluções comentadas.
 
 []'s Marcos Paulo 
 
 
 -- 
 Mensagens enviadas estão livres de vírus.
 Verificado por AVG Anti-Vírus (http://www.avgbrasil.com.br).
 Versão: 7.0.262 / Banco de dados de Vírus: 264.2.0 – Data de Lançamento: 2/8/2004
 
 
 
 =
 Instruções para entrar na lista, sair da lista e usar a lista em
 http://www.mat.puc-rio.br/~nicolau/olimp/obm-l.html
 =
 
 Esta mensagem foi verificada pelo E-mail Protegido Terra.
 Scan engine: VirusScan / Atualizado em 28/07/2004 / Versão: 1.5.2
 Proteja o seu e-mail Terra: http://www.emailprotegido.terra.com.br/
 


Fábio Bernardo
[EMAIL PROTECTED]
Tel. 2676-6854

[obm-l] Re: [obm-l] Questões estranhas

2004-08-03 Por tôpico Paulo Rodrigues
Essa prova do CN está esquisita mesmo. Vocês viram as questões 1 e 16?

No caso da 16, a resposta certa é a única que faz algum sentido, mas dá a
entender que toda seq. com uma quantidade limitada de valores é periódica!

Essas provas do CN já não foram melhores?

Paulo

- Original Message -
From: Marcos Paulo [EMAIL PROTECTED]
To: [EMAIL PROTECTED]
Sent: Tuesday, August 03, 2004 1:38 PM
Subject: Re: [obm-l] Questões estranhas


At 21:21 2/8/2004, you wrote:

Alguém poderia me dar uma ajuda nisso?

1 - Sabendo-se que a equação x^2*(x + 13) - 6x*(x^2 + 2) + 4 = 0 pode ser
escrita como o produto de dois binômios do primeiro grau, a soma de duas
das
suas raízes distintas é igual a:
Resp.: 3

Essa questão vc copiou errada a pergunta correta é:
Sabendo que x^2 (x^2 + 13) - 6x(x^2 + 2) + 4 = 0 pode ser escrita como o
produto de binomios do primeiro grau (e não de DOIS binomios)..

Note que o termo independente de x vale 4 e, portanto se houver alguma raiz
inteira essa será um dos divisores de 4, ou seja, +1, -1, +2, -2, +4, -4.
Basta testar 1 e 2 e vc verá que são raízes.

2 - O valor numérico da expressão 120x^4 + 10k^2 + 8, sendo k um natural, é
o quadrado de um número natural para:
Resp.: Nenhum valor de k


note que 120k^4 tem digito das unidades ZERO, asssim como 10k^2. POrtanto o
dígito das unidades da expressão inteira será 8 e não existe quadrado
perfeito que termine em 8, portanto independente do valor inteiro de k, a
expressão nunca será um quadrado perfeito.

Quem quiser ver a prova inteira do colégio naval pode entrar no endereço
www.cursoriachuelo.com.br/cn2004.htm neste endereço está o gabarito extra
oficial e clicando nas questões da prova azul abre uma janelinha com a
questão correspondente (enunciado + opções).
Se tiver um tempinho eu vou colocar em breve a prova com soluções
comentadas.

[]'s Marcos Paulo


--
Mensagens enviadas estão livres de vírus.
Verificado por AVG Anti-Vírus (http://www.avgbrasil.com.br).
Versão: 7.0.262 / Banco de dados de Vírus: 264.2.0 - Data de Lançamento:
2/8/2004



=
Instruções para entrar na lista, sair da lista e usar a lista em
http://www.mat.puc-rio.br/~nicolau/olimp/obm-l.html
=


---
Outgoing mail is certified Virus Free.
Checked by AVG anti-virus system (http://www.grisoft.com).
Version: 6.0.733 / Virus Database: 487 - Release Date: 2/8/2004

=
Instruções para entrar na lista, sair da lista e usar a lista em
http://www.mat.puc-rio.br/~nicolau/olimp/obm-l.html
=


Re: [obm-l] Re: [obm-l] Questões estranhas

2004-08-03 Por tôpico Marcos Paulo
Oi Paulo,
eu discordo da estranheza da prova. Achei que a prova foi bastante 
interessante ressucitando temas interessantes que estavam meio que às 
traças como o retângulo áureo (questão 7); o eixo radical (questão 17), a 
fórmula de transformação de radicais duplos em soma de radicais simples 
(questão 2). O produto notável pedido na questão 1 aparece em quase todo 
livro de oitava série (mesmo os piores) e eu não conheço outra 
justificativa (ou uma melhor) para que uma divisão entre inteiros resulte 
numa dízima periódica a não ser o fato de que exista uma quantidfade finita 
de restos possíveis na divisão, enquanto o processo (o algoritmo da 
divisão) pode ser repetido infinitamente.
Talvez a opinião dos outros membros da lista fosse interessante nessa questão.

[]'s MP
P.S. Os números das questões que eu citei são referentes à prova azul.
At 14:12 3/8/2004, you wrote:
Essa prova do CN está esquisita mesmo. Vocês viram as questões 1 e 16?
No caso da 16, a resposta certa é a única que faz algum sentido, mas dá a
entender que toda seq. com uma quantidade limitada de valores é periódica!
Essas provas do CN já não foram melhores?
Paulo

--
Mensagens enviadas estão livres de vírus.
Verificado por AVG Anti-Vírus (http://www.avgbrasil.com.br).
Versão: 7.0.262 / Banco de dados de Vírus: 264.2.0 – Data de Lançamento: 2/8/2004

=
Instruções para entrar na lista, sair da lista e usar a lista em
http://www.mat.puc-rio.br/~nicolau/olimp/obm-l.html
=


[obm-l] Re: [obm-l] Re: [obm-l] Questões estranhas

2004-08-03 Por tôpico Paulo Rodrigues
Acho que não fui claro...

Na questão 1 da prova está escrito:

Qual é o produto notável representado, geometricamente, na figura acima, na
qual ABCD é um retângulo?

A figura mosta um retângulo e tenta induzir   a expansão de
(a+b)^2=a^2+2ab+b^2.

Mas não existe produto notável representado na questão! Na figura não se faz
nem menção a área...

Posso estar sendo demais formal, porém, essa questão é imbecil para quem
conhece e indecifrável para quem não conhece.

Quanto a questão 16 você tem razão...interpretei quociente no lugar de
resto.

Em compensação a prova tem várias questões estranhas, que não levam a nada,
como a 4, que pergunta se é possível calcular a média aritmética, a média
geométrica e a harmônica, de dois números positivos.


- Original Message -
From: Marcos Paulo [EMAIL PROTECTED]
To: [EMAIL PROTECTED]
Sent: Tuesday, August 03, 2004 4:16 PM
Subject: Re: [obm-l] Re: [obm-l] Questões estranhas


Oi Paulo,
eu discordo da estranheza da prova. Achei que a prova foi bastante
interessante ressucitando temas interessantes que estavam meio que às
traças como o retângulo áureo (questão 7); o eixo radical (questão 17), a
fórmula de transformação de radicais duplos em soma de radicais simples
(questão 2). O produto notável pedido na questão 1 aparece em quase todo
livro de oitava série (mesmo os piores) e eu não conheço outra
justificativa (ou uma melhor) para que uma divisão entre inteiros resulte
numa dízima periódica a não ser o fato de que exista uma quantidfade finita
de restos possíveis na divisão, enquanto o processo (o algoritmo da
divisão) pode ser repetido infinitamente.
Talvez a opinião dos outros membros da lista fosse interessante nessa
questão.

[]'s MP

P.S. Os números das questões que eu citei são referentes à prova azul.

At 14:12 3/8/2004, you wrote:

Essa prova do CN está esquisita mesmo. Vocês viram as questões 1 e 16?

No caso da 16, a resposta certa é a única que faz algum sentido, mas dá a
entender que toda seq. com uma quantidade limitada de valores é periódica!

Essas provas do CN já não foram melhores?

Paulo


--
Mensagens enviadas estão livres de vírus.
Verificado por AVG Anti-Vírus (http://www.avgbrasil.com.br).
Versão: 7.0.262 / Banco de dados de Vírus: 264.2.0 - Data de Lançamento:
2/8/2004



=
Instruções para entrar na lista, sair da lista e usar a lista em
http://www.mat.puc-rio.br/~nicolau/olimp/obm-l.html
=


---
Outgoing mail is certified Virus Free.
Checked by AVG anti-virus system (http://www.grisoft.com).
Version: 6.0.733 / Virus Database: 487 - Release Date: 2/8/2004

=
Instruções para entrar na lista, sair da lista e usar a lista em
http://www.mat.puc-rio.br/~nicolau/olimp/obm-l.html
=


[obm-l] RE: [obm-l] Questões estranhas

2004-08-02 Por tôpico willian kanashiro


From: "Henrique Patrício Sant'Anna Branco" [EMAIL PROTECTED] 
Reply-To: [EMAIL PROTECTED] 
To: [EMAIL PROTECTED] 
Subject: [obm-l] Questões estranhas 
Date: Mon, 2 Aug 2004 21:21:20 -0300 
 
Alguém poderia me dar uma ajuda nisso? 
 
1 - Sabendo-se que a equação x^2*(x + 13) - 6x*(x^2 + 2) + 4 = 0 pode ser 
escrita como o produto de dois binômios do primeiro grau, a soma de duas das 
suas raízes distintas é igual a: 
Resp.: 3 
 
2 - O valor numérico da expressão 120x^4 + 10k^2 + 8, sendo k um natural, é 
o quadrado de um número natural para: 
Resp.: Nenhum valor de k 
 
Esse eu assumi que a equação pudesse ser fatorada como ((x - a)(x - b))^2 ou 
(x - c)^4 para ser um quadrado perfeito, resolvi a biquadrada e aí se chega 
à conclusão que não existe nenhuma raiz natural (nem mesmo real) dessa 
equação. É o modo certo de fazer? 
 
Grato, 
Henrique. 
 
= 
Instruções para entrar na lista, sair da lista e usar a lista em 
http://www.mat.puc-rio.br/~nicolau/olimp/obm-l.html 
= 

as duas questoes sao do colegio naval nao é??
eu sei pois prestei a prova tb
felizmente passei com 14 questoes

essas duas são faceis de resolver porem dificeis de demonstrar
entao irei fazer durante a semana essas questoes comentadas e mandarei assim que possivelMSN Messenger: converse com os seus amigos online.  Instale grátis. Clique aqui.  
=
Instruções para entrar na lista, sair da lista e usar a lista em
http://www.mat.puc-rio.br/~nicolau/olimp/obm-l.html
=


Re: [obm-l] Re: [obm-l] Questões confusas e vestibular do IME

2003-10-19 Por tôpico Alexandre Daibert
Caro colega,
para aplicarmos logaritmo nas equações teríamos que ter x e y reais e 
positivos. Não eh dado no problema nenhum dado como este. Logo, 
deveríamos considerar x e y no universo mais geral possível (no caso, os 
complexos...)
Não há também, como provar que x e y são reais positivos (a não ser q 
alguém aí consiga)
Por isso a minha crítica a esta questão, que na minha opinião é o maior 
desastre dos últimos 10 anos de prova do IME (pelo menos)

abraços,
Alexandre Daibert
[EMAIL PROTECTED] escreveu:

Caro amigo Alexandre Daibert , o vestibular do IME ( Instituto militar de
engenharia ) sem dúvida é o melhor do Brasil ; as provas são bem elaboradas
e procuram não só testar o conhecimento do aluno , com questõesde níveis
avançados , mais também a sagaciadade do mesmo , com questões de dupla interpretação
e/ou com várias soluções . Portanto , uma questão do IME não pode ser encarada
, por exemplo , como uma questão da AFA , onde um peixe que nada em direção
a superfície da água dentro de um aquário sobre uma balança , não altera
o medidor da mesma !
Vamos ao que interessa. No caso da questão enviada , temos :

CONSIDERE X e Y DIFERENTES DE ZERO !!!

x^ax = (ax)^x

Tirando log na base x nos dois lados da equação , temos :

logx x^ax = logx (ax)^x
ax = x( logx a + logx x )
a = logx a + 1 
a - 1 = logx a 

x^(a-1) = a 

x = a^(1/[a-1])

Agora é só substituir em y = ax e descobrir o valor de y em função de a
.
Abraços 
Luiz H. Barbosa 







--
Use o melhor sistema de busca da Internet
Radar UOL - http://www.radaruol.com.br


=
Instruções para entrar na lista, sair da lista e usar a lista em
http://www.mat.puc-rio.br/~nicolau/olimp/obm-l.html
=
 



=
Instruções para entrar na lista, sair da lista e usar a lista em
http://www.mat.puc-rio.br/~nicolau/olimp/obm-l.html
=


[obm-l] Re: [obm-l] Re: [obm-l] Questões confusas e vestibular do IME

2003-10-19 Por tôpico luizhenriquerick
Na verdade , eu acredito que este comentário feito por você :
...teríamos que ter x e y reais e 
positivos...
Deveria estar contido em sua solução !

Luiz H. Barbosa 


-- Mensagem original --

Caro colega,
para aplicarmos logaritmo nas equações teríamos que ter x e y reais e 
positivos. Não eh dado no problema nenhum dado como este. Logo, 
deveríamos considerar x e y no universo mais geral possível (no caso, os

complexos...)
Não há também, como provar que x e y são reais positivos (a não ser q 
alguém aí consiga)
Por isso a minha crítica a esta questão, que na minha opinião é o maior

desastre dos últimos 10 anos de prova do IME (pelo menos)

abraços,
Alexandre Daibert

[EMAIL PROTECTED] escreveu:

Caro amigo Alexandre Daibert , o vestibular do IME ( Instituto militar
de
engenharia ) sem dúvida é o melhor do Brasil ; as provas são bem elaboradas
e procuram não só testar o conhecimento do aluno , com questõesde níveis
avançados , mais também a sagaciadade do mesmo , com questões de dupla
interpretação
e/ou com várias soluções . Portanto , uma questão do IME não pode ser
encarada
, por exemplo , como uma questão da AFA , onde um peixe que nada em direção
a superfície da água dentro de um aquário sobre uma balança , não altera
o medidor da mesma !

Vamos ao que interessa. No caso da questão enviada , temos :

CONSIDERE X e Y DIFERENTES DE ZERO !!!

x^ax = (ax)^x

Tirando log na base x nos dois lados da equação , temos :

logx x^ax = logx (ax)^x
ax = x( logx a + logx x )
a = logx a + 1 
a - 1 = logx a 

x^(a-1) = a 
 
x = a^(1/[a-1])

Agora é só substituir em y = ax e descobrir o valor de y em função de
a
.

Abraços 
Luiz H. Barbosa 







--
Use o melhor sistema de busca da Internet
Radar UOL - http://www.radaruol.com.br



=
Instruções para entrar na lista, sair da lista e usar a lista em
http://www.mat.puc-rio.br/~nicolau/olimp/obm-l.html
=


  




=
Instruções para entrar na lista, sair da lista e usar a lista em
http://www.mat.puc-rio.br/~nicolau/olimp/obm-l.html
=




--
Use o melhor sistema de busca da Internet
Radar UOL - http://www.radaruol.com.br



=
Instruções para entrar na lista, sair da lista e usar a lista em
http://www.mat.puc-rio.br/~nicolau/olimp/obm-l.html
=


[obm-l] Re: [obm-l] Questões confusas e vestibular do IME

2003-10-18 Por tôpico luizhenriquerick
Caro amigo Alexandre Daibert , o vestibular do IME ( Instituto militar de
engenharia ) sem dúvida é o melhor do Brasil ; as provas são bem elaboradas
e procuram não só testar o conhecimento do aluno , com questõesde níveis
avançados , mais também a sagaciadade do mesmo , com questões de dupla interpretação
e/ou com várias soluções . Portanto , uma questão do IME não pode ser encarada
, por exemplo , como uma questão da AFA , onde um peixe que nada em direção
a superfície da água dentro de um aquário sobre uma balança , não altera
o medidor da mesma !

Vamos ao que interessa. No caso da questão enviada , temos :

CONSIDERE X e Y DIFERENTES DE ZERO !!!

x^ax = (ax)^x

Tirando log na base x nos dois lados da equação , temos :

logx x^ax = logx (ax)^x
ax = x( logx a + logx x )
a = logx a + 1 
a - 1 = logx a 

x^(a-1) = a 
 
x = a^(1/[a-1])

Agora é só substituir em y = ax e descobrir o valor de y em função de a
.

Abraços 
Luiz H. Barbosa 







--
Use o melhor sistema de busca da Internet
Radar UOL - http://www.radaruol.com.br



=
Instruções para entrar na lista, sair da lista e usar a lista em
http://www.mat.puc-rio.br/~nicolau/olimp/obm-l.html
=


[obm-l] Re:[obm-l] Questões da Olimpíada de Maio de 1999 (reenviada)

2003-09-11 Por tôpico claudio.buffara






De:
[EMAIL PROTECTED]




Para:
"OBM - Lista" <[EMAIL PROTECTED]>




Cópia:





Data:
Wed, 10 Sep 2003 20:30:11 -0300




Assunto:
[obm-l] Questões da Olimpíada de Maio de 1999 (reenviada)












Problema 3
A primeira fileira da tabela abaixo se preenche com os números de 1 a 10, em ordem crescente.
 [ ] [ ] [ ] [ ] [ ] [ ] [ ] [ ] [ ] [ ] [ ]
 [ ] [ ] [ ] [ ] [ ] [ ] [ ] [ ] [ ] [ ] [ ]
 [ ] [ ] [ ] [ ] [ ] [ ] [ ] [ ] [ ] [ ] [ ]
A segunda fileira se preenche com os números de 1 a 10, em qualquer ordem.
Em cada casa da terceira fileira se escreve a soma dos dois números escritos nas casas acima.
Existe alguma maneira de preencher a segunda fileira de modo que os algarismos das unidades dos números da terceira fileira sejam todos distintos?
Resposta: Não.

Seja S(k) = soma dos últimosalgarismos da k-ésima linha (k = 1, 2, 3).

Então S(1) = S(2) = 1 + 2 + ... + 9 + 10 = 55.
Logo: S(1) = S(2) == 5 (mod 10)

Além disso, vale S(3) == S(1) + S(2) (mod 10) == S(3) == 0 (mod 10).

Mas, se os algarismos das unidades da 3a. linha forem todos distintos, então teremos também S(3) = 55 == 5 (mod 10) == contradicão.

Um abraco,
Claudio.

[obm-l] RE: [obm-l] Questões da Olimpíada de Maio de 1999 (reenviada)

2003-09-11 Por tôpico João Gilberto Ponciano Pereira
Questão 1:
Um número natural de três algarismos é chamado de tricúbico se é igual a
soma dos cubos dos seus dígitos. Encontre todos os pares de números
consecutivos tais que ambos sejam tricúbicos.
100x + 10y + z = x3 + y3 + z3 
e
100x + 10y + (z+1) = x3 + y3 + (z+1)3
Subtraindo uma da outra e desenvolvendo, temos
z2 + z=0, logo z = 0 (não pode ser negativo)
 
Logo, x3 + y3 é divisível por 10. Analisando os cubos módulo 10 obtemos que
y = x-10
Logo,
100x + 10*(x-10) = x3 + (x-10)^3
x^2 - 13x + 30=0
x = 3 ou x = 10 (não vale)
 
logo, os únicos tricubicos consecutivos são 370 e 371

-Original Message-
From: Rodrigo Maranhão [mailto:[EMAIL PROTECTED]
Sent: Wednesday, September 10, 2003 8:30 PM
To: OBM - Lista
Subject: [obm-l] Questões da Olimpíada de Maio de 1999 (reenviada)



 

Estou reenviando o e-mail pq acho q o Server da lista não o encaminhou já q
estava com figura.




 

Abaixo vão dois problemas da olimpíada de maio de 1999 que eu gostaria de
saber as respostas:

Obs: O problema 1 eu resolvi e achei apenas 1 par de tricúbicos
consecutivos: 370 e 371. No entanto gostaria de confirmar se a resposta é
essa.

 

Problema 1

Um número natural de três algarismos é chamado de tricúbico se é igual a
soma dos cubos dos seus dígitos. Encontre todos os pares de números
consecutivos tais que ambos sejam tricúbicos.

 

Problema 3

A primeira fileira da tabela abaixo se preenche com os números de 1 a 10, em
ordem crescente.

[ ] [ ] [ ] [ ] [ ] [ ] [ ] [ ] [ ] [ ] [ ]

[ ] [ ] [ ] [ ] [ ] [ ] [ ] [ ] [ ] [ ] [ ]

[ ] [ ] [ ] [ ] [ ] [ ] [ ] [ ] [ ] [ ] [ ]

A segunda fileira se preenche com os números de 1 a 10, em qualquer ordem.

Em cada casa da terceira fileira se escreve a soma dos dois números escritos
nas casas acima.

Existe alguma maneira de preencher a segunda fileira de modo que os
algarismos das unidades dos números da terceira fileira sejam todos
distintos?

 

=
Instruções para entrar na lista, sair da lista e usar a lista em
http://www.mat.puc-rio.br/~nicolau/olimp/obm-l.html
=


[obm-l] Re: [obm-l] Questões da Olimpíada de Maio de 1999

2003-09-10 Por tôpico Domingos Jr.



faça assim,seja n = 100a + 10b + c = a³ + b³ 
+ c³(a, b, c dígitos, a  0)
casoc = 8
 n + 1 = 100a + 10b + c + 1 = a³ 
+ b³ + (c+1)³
 = 3c² + 3c = 0 = 
c = 0
 n = 100a + 10b, 10 | (a³ + b³ 
)

note que (1³, 2³, 3³, ..., 9³) = (1, 8, 7, 4, 5, 6, 
3, 2, 9)mod 10
ou seja, se fixarmos um valor para b, só há um 
valor que possa satisfazer a congruência módulo 10, então precisamos testar não 
mais do que dez pares de valores a, b...
um ex. pra vc pegar a idéia: se testarmos b = 2, 
então a = 8, pois 8³ + 2³ = 0 mod 10, mas 820 != 8³ + 2³ ... 
já b = 7 nos força a = 3 e 370 = 3³ + 
7³.

para o caso c = 9 devemos ter:
 b  9, se não a³ + b³ + c³ 
 2*9³  1000
 logo n = 100a + 10b + 9 = a³ + 
b³ + 9³
 en + 1 = 100a + 10(b + 1) 
= a³ + (b + 1)³ = n - 9³ + 3b² + 3b + 1 = 3b² + 3b = 9³, 
mas
 3b² + 3b  330  9³, logo 
não há pares consecutivos tricúbicos aqui...

[ ]'s


  
  
  Abaixo vão dois problemas da 
  olimpíada de maio de 1999 que eu gostaria de saber as 
  respostas:
  Obs: O problema 1 eu resolvi e 
  achei apenas 1 par de tricúbicos consecutivos: 370 e 371. No entanto gostaria 
  de confirmar se a resposta é essa.
  
  Problema 1
  Um número natural de três 
  algarismos é chamado de tricúbico se é igual a soma dos cubos 
  dos seus dígitos. Encontre todos os pares de números consecutivos tais que 
  ambos sejam tricúbicos.
  
  


[obm-l] Re: [obm-l] questões

2003-08-23 Por tôpico Fabio Bernardo
Suponhamos que todos tenham entrado com K reais. Assim.
A ficou 24 meses
B ficou 24 meses
C ficou 21 meses
D ficou 14 meses

24K+24K+21K+14K = 227835
83K=227835
K=2745

Então:
A receberá 65880
B receberá 65880
C receberá 57645
D receberá 38430

Espero ter ajudado!
Um abraço.

- Original Message -
From: elton francisco ferreira [EMAIL PROTECTED]
To: [EMAIL PROTECTED]
Sent: Saturday, August 23, 2003 10:02 AM
Subject: [obm-l] questões


Os valores de K tais que o sistema homogêneo  x + y +
2z = 0  admita apenas a solução trival, são
X + ky + z = 0
Kx - y - z = 0

a) k ¹ 0 e k ¹ -1
b) k ¹ 1 e k ¹ -1
c) k = 0 e k = -1
d) k ¹ 1 e k ¹ -2


Sobre a equação 1983x^2 - 1984x - 1985 = 0, a
afirmação correta é:

a) não tem raízes reais
b) tem duas raízes simétricas
c) tem duas raízes reais distintas
d) tem duas raízes reais iguais

As promoções do tipo  leve 3, pague 2, comuns no
comércio, acenam um desconto, sobre cada unidade
vendida, de

a) 50/3%
b) 100/3%
c) 20%
d) 50%











Duas pessoas A e B fundaram uma sociedade. Três meses
depois, admitiram outro sócio C. sete meses depois da
entrada de C, aceitaram outro sócio D. essas 4 pessoas
entraram para a sociedade com capitais iguais. Dois
anos após a fundação da sociedade, foi verificado um
lucor de 227.835,00. se este lucro foi dividido entre
os sócios, proporcionalmente ao tempo de participação
de cada um na sociedade, a parte que coube ao sócio D
foi:

57.645
38.430
21.352
18.234


___
Desafio AntiZona: participe do jogo de perguntas e respostas que vai
dar um Renault Clio, computadores, câmeras digitais, videogames e muito
mais! www.cade.com.br/antizona
=
Instruções para entrar na lista, sair da lista e usar a lista em
http://www.mat.puc-rio.br/~nicolau/olimp/obm-l.html
=

Esta mensagem foi verificada pelo E-mail Protegido Terra.
Scan engine: VirusScan / Atualizado em 20/08/2003 / Versão: 1.3.13
Proteja o seu e-mail Terra: http://www.emailprotegido.terra.com.br/




=
Instruções para entrar na lista, sair da lista e usar a lista em
http://www.mat.puc-rio.br/~nicolau/olimp/obm-l.html
=


[obm-l] Re: [obm-l] Questões Divertidas

2003-08-20 Por tôpico Frederico Reis Marques de Brito
Olá Cláudio ( obrigado por ter dado atenção às minhas questoes) e demais 
COLEGAS da lista ( por colegas entendo aqueles que, de alguma forma, estão 
realmenteinteressados na discussão sobre a Matemática e suas belezas 
contribuindo efetivamente para a manutenção e o desenvolvimento da cultura 
matemática neste país.).

Correto. Concordo com as três soluções. Entretanto para o segundo exercício 
podemos dar uma solução mais rápida:
como   a^2b^2c^2 + ab +ac + bc = wabcpara todo  a, b, c positivos , 
fazendo a=b=c=1, temos:
w=4 .
Resta provar que  w=4 satisfaz a condição imposta no enunciado. Para tanto, 
usamos novamente, a desigualdade entere as médias,  MA = MG:
(a^2b^2c^2+ab+ac+bc)/4 =  (a^2b^2c^2abacbc)^{1/4} = (a^4b^4c^4)^{1/4}=abc 
=
(abc)^2+ab+ac+bc = 4abc.

Um grande abraço,
Frederico.

From: Claudio Buffara [EMAIL PROTECTED]
Reply-To: [EMAIL PROTECTED]
To: [EMAIL PROTECTED]
Subject: Re: [obm-l] Questões Divertidas
Date: Tue, 19 Aug 2003 15:08:27 -0300
Oi, Frederico:

Jah que ninguem mais respondeu, aqui vai...

 (1)Mostre  que  tg(x)  + cotg (x) = 2

Supondo que x (mod 2Pi) esteja em (0,Pi/2) U (Pi,3Pi/2), o resultado eh
consequencia de que (tg(x) - 1)^2 = 0.

 (2)  Encontre o maior número real   w tal que wabc =   (abc)^2 
+ ab
 + ac + bc , para todo a,b,c 0 .

O problema equivale a achar o valor minimo de:
F(a,b,c) = abc + 1/a + 1/b + 1/c, com a,b,c  0.

Esse deu um certo trabalho, mas consegui descobrir uma solucao sem usar
calculo.
Media Geometrica = Media Harmonica ==
(abc)^(1/3) = 3/(1/a + 1/b + 1/c) ==
abc = 27/(1/a + 1/b + 1/c)^3 ==
F(a,b,c) = 27/(1/a +1/b + 1/c)^3 + (1/a + 1/b + 1/c),
com igualdade == a = b = c, ou seja:
F(a,b,c) eh minimo quando a = b = c
Mas, fazendo x = 1/a + 1/b + 1/c, teremos:
F(a,b,c) = 27/x^3 + x = 4*[27/x^3 + x/3 + x/3 + x/3]/4
Media Aritmetica = Media Geometrica ==
[27/x^3 + x/3 + x/3 + x/3]/4 = [(27/x^3)*(x/3)*(x/3)*(x/3)]^(1/4) = 1 ==
27/x^3 + x/3 + x/3 + x/3 = 27/x^3 + x = 4,
com igualdade == 27/x^3 = x/3 == x = 3 == 1/a + 1/b + 1/c = 3, ou
seja:
F(a,b,c) eh minimo quando 1/a + 1/b + 1/c = 3.
Assim, o valor minimo de F(a,b,c) eh atingido quando:
a = b = c e 1/a + 1/b + 1/c = 3 == a = b = c = 1
e nesse caso F(a,b,c) = 4
Conclusao: o maior w eh igual a 4.



 (3)  V ou F:O produto da soma de nos reais positivos pela soma de 
seus
 inversos é =  ao quadrado da quantidade de números.

V - consequencia da desigualdade entre a media harmonica e a media
geometrica de numeros positivos.
Um abraco,
Claudio.
=
Instruções para entrar na lista, sair da lista e usar a lista em
http://www.mat.puc-rio.br/~nicolau/olimp/obm-l.html
=
_
MSN Messenger: converse com os seus amigos online.  
http://messenger.msn.com.br

=
Instruções para entrar na lista, sair da lista e usar a lista em
http://www.mat.puc-rio.br/~nicolau/olimp/obm-l.html
=


[obm-l] Re: [obm-l] Re: [obm-l] Questões Divertidas

2003-08-19 Por tôpico Frederico Reis Marques de Brito
Olá a todos. Realmente, este fato só é válido nos quadrantes ímpares. Achei 
que já tinha mandado esta errata para a lista mas pelo visto devo ter 
respondido a algum email pessoal. De qq forma, obrigado Morgado.
Abraços,
Frederico.


From: Augusto Cesar de Oliveira Morgado [EMAIL PROTECTED]
Reply-To: [EMAIL PROTECTED]
To: [EMAIL PROTECTED]
Subject: Re: [obm-l] Re: [obm-l] Questões Divertidas
Date: Mon, 18 Aug 2003 18:54:42 -0300 (EST)
Epa, isso so eh verdade no primeiro e no terceiro quadrantes.
Morgado
Em Mon, 18 Aug 2003 15:33:52 -0400, Aleandre Augusto da Rocha 
[EMAIL PROTECTED] disse:


 - Original Message -
 From: Frederico Reis Marques de Brito [EMAIL PROTECTED]
 To: [EMAIL PROTECTED]
 Sent: Saturday, August 16, 2003 9:47 AM
 Subject: [obm-l] Questões Divertidas


 
  Caros colegas. As questões que se seguem são todas simples, desde que
  pensemos na coisa certa. Como gostei delas resolvi partilhá-las com 
vcs:
 
  (1)Mostre  que  tg(x)  + cotg (x) = 2  .
 

 tg(x) + cotg(x) = 2
 sen(x)/cos(x) + cos(x)/sen(x) = 2
 (sen^2(x) + cos^2(x))/sen(x)cos(x) = 2
 1 = 2sen(x)cos(x)
 1=sen(2x)

 
  Abraços,
  Frederico.

 -Auggy


 
=
 Instruções para entrar na lista, sair da lista e usar a lista em
 http://www.mat.puc-rio.br/~nicolau/olimp/obm-l.html
 
=



=
Instruções para entrar na lista, sair da lista e usar a lista em
http://www.mat.puc-rio.br/~nicolau/olimp/obm-l.html
=
_
MSN Messenger: converse com os seus amigos online.  
http://messenger.msn.com.br

=
Instruções para entrar na lista, sair da lista e usar a lista em
http://www.mat.puc-rio.br/~nicolau/olimp/obm-l.html
=


[obm-l] Re: [obm-l] Re: [obm-l] Questões Divertidas

2003-08-19 Por tôpico Frederico Reis Marques de Brito
Olá Alexandre. É bastante perigoso  tentarmos demonstrar uma 
desigualdadeou mesmo uma igualdade mechendo nos dois membros da mesma. Ao 
chegarmos numa conclusão verdadeira, como a que vc chegou, é necessário 
checar se os passos são treversíveis, pois afinal de contas vc partiu da 
hipótese.  Além disso, senx . cos x pode ser negativo , o que ocorre 
precisamente nos quadrantes pares, onde a tese é falsa, mas  vc não se deu 
conta disso.
Uma sugestão, olhe os outros problemas e tente achar a idéia comum aos três;

Um abraço,
Frederico.

From: Aleandre Augusto da Rocha [EMAIL PROTECTED]
Reply-To: [EMAIL PROTECTED]
To: [EMAIL PROTECTED]
Subject: [obm-l] Re: [obm-l] Questões Divertidas
Date: Mon, 18 Aug 2003 15:33:52 -0400
- Original Message -
From: Frederico Reis Marques de Brito [EMAIL PROTECTED]
To: [EMAIL PROTECTED]
Sent: Saturday, August 16, 2003 9:47 AM
Subject: [obm-l] Questões Divertidas

 Caros colegas. As questões que se seguem são todas simples, desde que
 pensemos na coisa certa. Como gostei delas resolvi partilhá-las com 
vcs:

 (1)Mostre  que  tg(x)  + cotg (x) = 2  .


tg(x) + cotg(x) = 2
sen(x)/cos(x) + cos(x)/sen(x) = 2
(sen^2(x) + cos^2(x))/sen(x)cos(x) = 2
1 = 2sen(x)cos(x)
1=sen(2x)

 Abraços,
 Frederico.
-Auggy

=
Instruções para entrar na lista, sair da lista e usar a lista em
http://www.mat.puc-rio.br/~nicolau/olimp/obm-l.html
=
_
MSN Messenger: converse com os seus amigos online.  
http://messenger.msn.com.br

=
Instruções para entrar na lista, sair da lista e usar a lista em
http://www.mat.puc-rio.br/~nicolau/olimp/obm-l.html
=


Re: [obm-l] Re: [obm-l] Re: [obm-l] Questões Divertidas

2003-08-19 Por tôpico Johann Peter Gustav Lejeune Dirichlet
Tres coisas:
1)O enunciado nao dizia sobre os quadrantes,logo
esta errado!
2)voce queria que ele resolvesse como???E claro
que ele tem que gatrantir que as passagens sao
equivalencias.Ou se voce nao viu o problema
começaria diferente se usassemos a desigualdade
das medias?
3)mecher e nao MEXER

 --- Frederico Reis Marques de Brito
[EMAIL PROTECTED] escreveu:  Olá
Alexandre. É bastante perigoso  tentarmos
 demonstrar uma 
 desigualdadeou mesmo uma igualdade mechendo
 nos dois membros da mesma. Ao 
 chegarmos numa conclusão verdadeira, como a que
 vc chegou, é necessário 
 checar se os passos são treversíveis, pois
 afinal de contas vc partiu da 
 hipótese.  Além disso, senx . cos x pode ser
 negativo , o que ocorre 
 precisamente nos quadrantes pares, onde a tese
 é falsa, mas  vc não se deu 
 conta disso.
 Uma sugestão, olhe os outros problemas e tente
 achar a idéia comum aos três;
 
 
 Um abraço,
 Frederico.
 
 
 From: Aleandre Augusto da Rocha
 [EMAIL PROTECTED]
 Reply-To: [EMAIL PROTECTED]
 To: [EMAIL PROTECTED]
 Subject: [obm-l] Re: [obm-l] Questões
 Divertidas
 Date: Mon, 18 Aug 2003 15:33:52 -0400
 
 
 - Original Message -
 From: Frederico Reis Marques de Brito
 [EMAIL PROTECTED]
 To: [EMAIL PROTECTED]
 Sent: Saturday, August 16, 2003 9:47 AM
 Subject: [obm-l] Questões Divertidas
 
 
  
   Caros colegas. As questões que se seguem
 são todas simples, desde que
   pensemos na coisa certa. Como gostei
 delas resolvi partilhá-las com 
 vcs:
  
   (1)Mostre  que  tg(x)  + cotg (x) = 2 
 .
  
 
 tg(x) + cotg(x) = 2
 sen(x)/cos(x) + cos(x)/sen(x) = 2
 (sen^2(x) + cos^2(x))/sen(x)cos(x) = 2
 1 = 2sen(x)cos(x)
 1=sen(2x)
 
  
   Abraços,
   Frederico.
 
 -Auggy
 
 

=
 Instruções para entrar na lista, sair da lista
 e usar a lista em

http://www.mat.puc-rio.br/~nicolau/olimp/obm-l.html

=
 

_
 MSN Messenger: converse com os seus amigos
 online.  
 http://messenger.msn.com.br
 

=
 Instruções para entrar na lista, sair da lista
 e usar a lista em

http://www.mat.puc-rio.br/~nicolau/olimp/obm-l.html

= 

___
Desafio AntiZona - Um emocionante desafio de perguntas e respostas que
te dá um Renault Clio, kits de eletrônicos, computadores, notebooks e 
mochilas. Cadastre-se, participe e concorra!
www.cade.com.br/antizona
=
Instruções para entrar na lista, sair da lista e usar a lista em
http://www.mat.puc-rio.br/~nicolau/olimp/obm-l.html
=


[obm-l] Re: [obm-l] Questões Divertidas

2003-08-18 Por tôpico Aleandre Augusto da Rocha

- Original Message -
From: Frederico Reis Marques de Brito [EMAIL PROTECTED]
To: [EMAIL PROTECTED]
Sent: Saturday, August 16, 2003 9:47 AM
Subject: [obm-l] Questões Divertidas



 Caros colegas. As questões que se seguem são todas simples, desde que
 pensemos na coisa certa. Como gostei delas resolvi partilhá-las com vcs:

 (1)Mostre  que  tg(x)  + cotg (x) = 2  .


tg(x) + cotg(x) = 2
sen(x)/cos(x) + cos(x)/sen(x) = 2
(sen^2(x) + cos^2(x))/sen(x)cos(x) = 2
1 = 2sen(x)cos(x)
1=sen(2x)


 Abraços,
 Frederico.

-Auggy


=
Instruções para entrar na lista, sair da lista e usar a lista em
http://www.mat.puc-rio.br/~nicolau/olimp/obm-l.html
=


Re: [obm-l] Re: [obm-l] Questões Divertidas

2003-08-18 Por tôpico Augusto Cesar de Oliveira Morgado
Epa, isso so eh verdade no primeiro e no terceiro quadrantes.
Morgado

Em Mon, 18 Aug 2003 15:33:52 -0400, Aleandre Augusto da Rocha [EMAIL PROTECTED] 
disse:

 
 - Original Message -
 From: Frederico Reis Marques de Brito [EMAIL PROTECTED]
 To: [EMAIL PROTECTED]
 Sent: Saturday, August 16, 2003 9:47 AM
 Subject: [obm-l] Questões Divertidas
 
 
 
  Caros colegas. As questões que se seguem são todas simples, desde que
  pensemos na coisa certa. Como gostei delas resolvi partilhá-las com vcs:
 
  (1)Mostre  que  tg(x)  + cotg (x) = 2  .
 
 
 tg(x) + cotg(x) = 2
 sen(x)/cos(x) + cos(x)/sen(x) = 2
 (sen^2(x) + cos^2(x))/sen(x)cos(x) = 2
 1 = 2sen(x)cos(x)
 1=sen(2x)
 
 
  Abraços,
  Frederico.
 
 -Auggy
 
 
 =
 Instruções para entrar na lista, sair da lista e usar a lista em
 http://www.mat.puc-rio.br/~nicolau/olimp/obm-l.html
 =
 
 

=
Instruções para entrar na lista, sair da lista e usar a lista em
http://www.mat.puc-rio.br/~nicolau/olimp/obm-l.html
=


[obm-l] Re: [obm-l] Re:[obm-l] Re: [obm-l] Questões Esaex - RETA FINAL 4 e 5

2003-08-14 Por tôpico J Augusto Tavares
Obrigado... eu nao formatei o texto ... os simbolos q tao com problemas sao
os da msg original!!
eu copiei o simbolo de infinito pra colocar na minha msg, ele deu errado
vou colocar so as respostas da 4 e 5 cujo titulo eh
[obm-l] Questões Esaex - RETA FINAL  do Joao!

- Original Message -
From: amurpe [EMAIL PROTECTED]
To: [EMAIL PROTECTED]
Cc: [EMAIL PROTECTED]
Sent: Thursday, August 14, 2003 4:43 AM
Subject: [obm-l] Re:[obm-l] Re: [obm-l] Questões Esaex - RETA FINAL 4 e 5


 
 Os simbolos estão com problema, por favor reenvie a msg.

 Um abraço.Amurpe
4) NÃfO CONSIGO FATORAR A RAIZ CÃsBICA!

Resposta:
 
Fazendo  (x+1) = y^12 , como x-0  ,   y-1.
 
 (y^3 + y^4 + y^6 - 3)/(y^6 - 1) ,  [(y^3 -
  1) + (y^4 -1) + (y^6 - 1)]/[(y^3 + 1)(y^3 -1)]
 
 eleminando o fator (y- 1), nao existira mais  a indeterminacao !
 
 
 
 
 
 
 
5)

Resposta:
 
n-+infinito
 
ln[lim (1 + 2/(x+3))^(2x - 1/3)], lim (1 + 2/(x+3))^
 (x+3)*[(2x -1/3)/(x+3)  Elevando a (x+3)/(x+3)
 
temos o limite fundamental { n-+infinito } d  (1 + k/x)^x = e^k
 
(e^2)^lim(2x -1/3)/(x+3) = (e^2)^2 , logo ln e^4  eh 4.
 
 
 
 
 
 
 
 


 __
 Acabe com aquelas janelinhas que pulam na sua tela.
 AntiPop-up UOL - É grátis!
 http://antipopup.uol.com.br/


 =
 Instruções para entrar na lista, sair da lista e usar a lista em
 http://www.mat.puc-rio.br/~nicolau/olimp/obm-l.html
 =


=
Instruções para entrar na lista, sair da lista e usar a lista em
http://www.mat.puc-rio.br/~nicolau/olimp/obm-l.html
=


[obm-l] Re:[obm-l] Re: [obm-l] Questões Esaex - RETA FINAL 4 e 5

2003-08-14 Por tôpico amurpe
 
Os simbolos estão com problema, por favor reenvie a msg.

Um abraço.Amurpe



   1) Seja:
 
   Z1 = 2eiπ/6Z2 = (1 + i/1 – i)
15ln Z3 = 1 + i(π/3 + 2kπ)
 

 
   Então o valor de (Z1 +  Z2)/e  . Z3  é:
 
   R: minha resposta deu diferente da do fabio ... como 
ninguem falou nada ate agora ! a minha deve ta errada!
 

 
 
 
   4) NÃO CONSIGO FATORAR A RAIZ CÚBICA!
 
   O valor do limite quando x → 0 de
 

  4√(x+1) + 3√(x+1) + √(x+1) – 3
 

√(x+1) – 1
 

 
 
 
   Resposta: 
 
   Fazendo  (x+1) = y^12 , como x-0  ,   y-1.
 
(y^3 + y^4 + y^6 - 3)/(y^6 - 1) ,  [(y^3 -
 1) + (y^4 -1) + (y^6 - 1)]/[(y^3 + 1)(y^3 -1)]
 
eleminando o fator (y-
1), nao existira mais  a indeterminacao ! 
 

 

 
 
 
   5) Determinar lim   ln (1 +  _2_  )2x – 1/3  
 
  n→∞x+3

 
 
 
   Resposta:
 
   n-∞
 
   ln[lim (1 + 2/(x+3))^(2x - 1/3)], lim (1 + 2/(x+3))^
(x+3)*[(2x -1/3)/(x+3)  Elevando a (x+3)/(x+3)
 
   temos o limite fundamental { n-∞ }  (1 + k/x)
^x = e^k
 
   (e^2)^lim(2x -1/3)/(x+3) = (e^2)
^2 , logo ln e^4  eh 4.
 

 
 
 

 
 

 
__
Acabe com aquelas janelinhas que pulam na sua tela.
AntiPop-up UOL - É grátis!
http://antipopup.uol.com.br/


=
Instruções para entrar na lista, sair da lista e usar a lista em
http://www.mat.puc-rio.br/~nicolau/olimp/obm-l.html
=


[obm-l] Re: [obm-l] Questões da ESAEx Por favor!

2003-08-14 Por tôpico Cláudio \(Prática\)



Ficaram faltando estas três:

3) O produto das distâncias de um ponto qualquer de 
uma hipérbole de equação
 (x^2/a^2) - (y^2/b^2) = 1 às suas 
assíntotas é ?

Assíntotas: 
1) ay- bx = 
0 
2) ay+ bx = 0.

Seja o ponto P = (r,s) pertencente à 
hipérbole.
A distancia de P até a assíntota 1 é (fórmula da 
distância de ponto a reta):
| as - br |/raiz(a^2+b^2)

A distancia de P até a assíntota2 
é:
| as+ br |/raiz(a^2+b^2)

Portanto, Produto = |a^2s^2 - 
b^2r^2|/(a^2+b^2)

Mas P pertence à hipérbole. Logo:
r^2/a^2 - s^2/b^2 = 1 ==
b^2r^2 - a^2s^2 = a^2b^2 ==
|a^2s^2 - b^2r^2| = a^2b^2 ==

Produto = a^2b^2/(a^2+b^2) = 
constante.

*

7) Considerando um sistema linear de 10 equações e 
10 incógnitas, o número de multiplicações e divisões necessárias para resolvê-lo pela regra de Cramer é igual a ?

Serão calculados 11 determinantes 10x10 e depois 
serão necessarias 10 divisões (uma pra cada variável).
Cada determinante tem 10! termos e cada termo 
envolve o produto de 10 numeros == 
cada determinante envolve 10*10! multiplicações 
== 
o numero total de multiplicações será 11*10*10! = 
10*11!

Assim, serão efetuadas 10*11! multiplicações e 11 
divisões.

Moral da história: use eliminação 
gaussiana.

*

9) A matriz da transformação que passa xy = 1 para 
a forma canônica (x^2) / 2 + (y^2)/2 = 1 é ?

xy = 1 é equação de uma hipérbole equilátera tendo 
os eixos coordenados como assíntotas, com semi-eixos medindo raiz(2) e focos 
sobre a reta y = x;

x^2/2 + y^2/2 = 1 é equação de uma circunferência 
de raio raiz(2) centrada na origem. 

Assim, não existe nenhuma transformação linear que 
faça esta passagem.

Por outro lado, se a segunda equação fosse x^2/2 - 
y^2/2 = 1, ela representaria uma hipérbole equilátera cujas assíntotas são as 
retas y = x e y = -x, com semi-eixos medindo raiz(2) e focos sobre o 
eixo-x.

Nesse caso, a transformação seria uma rotação de 
-pi/4 (ou 7pi/4) em torno da origem.
Matriz =|cos(-pi/4) 
sen(-pi/4) | = (1/raiz(2)) * | 1 -1 |
 
| -sen(-pi/4) cos(-pi/4) 
| 
| -1 1 |


Um abraço,
Claudio.


[obm-l] Re: [obm-l] Questões da ESAEx Por favor!

2003-08-03 Por tôpico luizhenriquerick
2) lim(e^t - cost - sent)/t^2 
  t-0


Olá , se eu entendi bem o enunciado da questão , ai vai a solução:

Quando substituímos t por 0 na expressão , verificamos o caso de indeterminação
0/0 , com isso , podemos derivar o numerador e o denominador para que o
caso de indeterminação suma .

[e^t + sent - cost]/2t

Substituindo t por 0 novamente , o caso de indeterminação ainda figura na
expressão , assim , repetimos o mesmo processo:

[e^t + cost + sent]/2 

Então , finalmente , quando substituímos t por 0 , fica:

[e^t + cost + sent]/2 = [1 + 1 + 0]/2 = 1


Tente fazer a outra de limites , usando um limite fundamental .As de somatório
, pense mais um pouco , vale a pena .

Abraço.

Luiz H. Barbosa 



 


 

www.olympicmaths.hpg.com.br


--
Use o melhor sistema de busca da Internet
Radar UOL - http://www.radaruol.com.br



=
Instruções para entrar na lista, sair da lista e usar a lista em
http://www.mat.puc-rio.br/~nicolau/olimp/obm-l.html
=


[obm-l] Re: [obm-l] QUESTÕES INTERESSANTES

2003-07-09 Por tôpico yurigomes
Oi Frederico,
 Gostei das questões! =P

(1)( = ) Suponha A e A^(-1) com entradas inteiras. Então  detA e detA^(-1)
são inteiros. Mas como detA.detA^(-1)= 1, devemos ter detA= +-1.
 ( = ) Ora, se A= (a  b), então A^(-1)= 1/detA.(d  -b), e assim 
   (c  d)   (-c  a)
como detA= +-1, segue que A^(-1) tem entradas inteiras.

(2) Se vc tem um complexo z= a+ bi, então a rotação de z sobre a origem
de um ângulo A é obtido multiplicando z por cisA= cosA + i.senA. Logo os
outros dois vértices do triângulo são:
(sqrt(3)+ i)cis(120º) e (sqrt(3)+ i)cis(240º).

(3) Temos raiz real dupla se e só se delta= 0 =
  b^2= 4ac. Daí sendo b primo, temos b= 2, e assim 2^2= 4ac =
1= ac. Logo a, c não podem ser primos.

(4) Basta analisar módulo 8. Os quadrados só deixam resto 0, 1 e 4 (módulo
8), Mas 3^x= 1 (mod 8) se x é par e 3^x= 3 (mod 8) se x é ímpar. Logo 3^a+
3^b+ 1 só pode deixar resto
 1+ 1+ 1= 3 (mod 8)
 1+ 3+ 1= 5 (mod 8)
 3+ 3+ 1= 7 (mod 8)
 Assim, 3^a+ 3^b+ 1 nunca pode ser um quadrado perfeito.

(5) Se 1/p= a_0,a_1...a_i b_1...b_T(p) b_1...b_T(p)..., então
   10^i.(1/p)=a_0 a_1...a_i,b_1..b_T(p) b_1..b_T(p)..
10^(i+T(p))(1/p)=a_0 a_1..a_ib_1..b_T(p),b_1..b_T(p)...
 Daí,
(10^(i+T(p))-10^i).(1/p)=k, onde k é inteiro. Logo:
   (10^i)(10^(T(p))- 1)= p.k
  Como p é maior do que 5, temos (p, 10^i)= 1, donde p|(10^T(p)- 1) = 10^(T(p))=
1 (mod p) (*)
 (a) Se T(p)=2, então p|10^2- 1= 99 = p= 3 ou p= 11. Como p5, temos que
p=11.
 (b) Continuando de (*), temos ord10(mod p)|T(p). Mas é fácil fazer o mesmo
que fizemos acima voltando para concluir que ord10(mod p)= T(p). Daí, T(p)|(p-1),
pois 10^(p- 1)= 1 (mod p), Além disso, R(n)= 111= 1+ 10+ ...+ 10^(n-
1)= (10^n- 1)/9. Logo p|R(n) = p|(10^n- 1)/9 = p|(10^n- 1) = 10^n=
1 (mod p), e o valor mínimo de n satisfazendo essa ultima congruencia é
ord10(mod p)= T(p).

(6) Se a parábola tivesse raizes racionais, deveriamos ter delta= x^2, com
x inteiro. Daí:
b^2- 4.ac= x^2 = x ímpar
 Então b^2= x^2= 1 (mod 8), de modo que 8|(b^2- x^2) = 8|4ac, absurdo,
já que a, c são impares.

   Ateh mais,
  Yuri
   Mensagem original --

Olá caros colegas da lista, seguem algumas questões que considero muito

interessantes, para quem quiser se distrair um pouco nessas férias:

(1)   Seja   A   uma matriz 2x2  com entradas inteiras. Mostre que  A tem

inversa  com entradas ingteiras   se, e somente se,   det(A) = + - 1
.
(
Uma das implicações é trivial. )

(2)   Um triângulo equilátero, inscrito numa circunferência de centro na

origem do sistema cartesiano, tem  o   número complexoz= sqrt(3)  +
i

como um de seus vértices. Determine os outros.

(3)  Uma equação quadrática com coeficientes primos pode ter raiz real

dupla?

(4) Sejam   a   , bnúmeros naturais.  Mostre  que (3^a  + 3^b + 1 )


nunca é um quadrado perfeito.

(5) Seja  p   5   um número primo. Neste caso   1/p , o recíproco de
p
,
tem por  representação decimal uma dízima periódica. Indiquemos por  T(p)
 o
número de algarismos que constituem o período.
(a)  Mostre que existe um único primo   p  tal que  T(p)  = 2 .
(b) Mostre que  T(p)   é o menor número natural  n   tal   que:  n
|

(p-1) e  p | R_n  ,  em que  R_n = ...1 ( n  dígitos 1 ) .
Obs:  a | b  quandob   é múltiplo de  a , isto é, existe  q  inteiro
tal
que   b = a . q   .

(6) Mostre que se   a , b, c  são inteiros  ímpares   então a eq. 
ax^2+bx+c=0   não tem raiz racional.
( Proposto por Eduardo Wagner no 1o encontro da RPM. )

Até a próxima.
Frederico.


From: Frederico Reis Marques de Brito [EMAIL PROTECTED]
Reply-To: [EMAIL PROTECTED]
To: [EMAIL PROTECTED]
Subject: Re: [obm-l] Duvida sobre polinomios
Date: Wed, 09 Jul 2003 11:08:23 -0300


Sim, desde que o polinômio divisor não seja nulo. Existe um resultado,

análogo ao Lema da Divisão de Euclides para nos inteiros, que garante
que

dados   polinõmios   f(x), g(x) , g(x)  0,  com coeficientes num corpo
K
( em particular se  K= R = conjunto dos numeros reais )  então existem
e

são únicos  q(x) e  r(x)  com coeficientes em  K  de tal forma que 
 f(x)

= g(x) X q(x) + r(x) e  r(x) =0   ougrau(r)  gr(g) .  A

demonstração desse fato é, normalmente, obtida através de indução
matemática. Você pode obtê-la, por exemplo, em

[1]GONÇALVES, Adilson -  Introdução Á Álgebra  -  Projeto Euclides
-
SBM/IMPA.
[2]   DOMINGUES, Hygino H. , IEZZI, Gelson -  Álgebra Moderna-  Atual

Editora.

Observe ainda que a hipótese de que os polinômios tenham coeficientes
num

corpo ( anel comutativo com elemento neutro do produto e no qual todo

elemento não -nulo tenha inverso, ufa!!! ) é absolutamente essencial.
SE

dividirmos  F(x)= x+1   por   G(x)  = 2   , olhando-os como polinômios
a

coef. inteiros, não obteremos um quociente com esta propriedade.
Frederico.

From: leonardo mattos [EMAIL PROTECTED]
Reply-To: [EMAIL PROTECTED]
To: [EMAIL PROTECTED]
Subject: [obm-l] Duvida sobre polinomios
Date: Wed, 09 Jul 2003 12:05:03 +

Gostaria 

[obm-l] Re:[obm-l] Questões da EsaEx

2003-07-06 Por tôpico thor-oliveira
 Acho que não é off-topic.
 Se for, me perdoem, mas questiono se alguém tem questõe
s ou provas de concursos anteriores
 da EsaEx ( Escola de Administração do Exército ) para a
 área Magistério - Matemática?
 
 FORTE ABRAÇO
 
 Ola Joao

   Entra no site (www.estudemais.com.br)la voce vai 
encontrar um CD Rom que tem essas provas.


  Um abraço.

   Cláudio Thor.

 
__
Acabe com aquelas janelinhas que pulam na sua tela.
AntiPop-up UOL - É grátis!
http://antipopup.uol.com.br/


=
Instruções para entrar na lista, sair da lista e usar a lista em
http://www.mat.puc-rio.br/~nicolau/olimp/obm-l.html
=


[obm-l] Re: [obm-l] questões

2002-10-04 Por tôpico leonardo mattos

Ola Rafael,
1)Multiplique todas a linhas por 1 e some à 1ªlinha.Coloque em evidencia o 
termo em comum da 1ªlinha.Tendo agora so termos 1 na 1ªlinha faça todas as 
colunas menos a 1ª.Calcule agora o det da matriz triangular formada.
2)Fatore 360 que vc encontrara todos os seus divisores naturais...
3)Deixo essa para ser ainda discutida pela lista,ja que o proprio criador da 
questao disse q essa questao nao era de combinatoria,mas sim teoria dos 
grafos...
 Um abraço,Leonardo

From: rafaelc.l [EMAIL PROTECTED]
Reply-To: [EMAIL PROTECTED]
To: [EMAIL PROTECTED]
Subject: [obm-l] questões
Date: Fri,  4 Oct 2002 21:35:37 -0300



  Por favor, me ajudem nas questões abaixo:

  1) Qual o determinante de uma matriz de ordem n que
possui zeros na diagonal principal e todos os outros
elementos iguais a 1?
  2)quantos são os divisores naturais de 360? qual sua
soma?
  3) Naquela questão 7 do IME 2001, das estradinhas, pq
estaria errado considerar o número de percursos com n
movimentos como: 3.2^n-2? da cidade A temos 3
posssibilidades e depois de cada cidade temos duas
possibilidades( menos a cidade A).



obrigado


__
Encontre sempre uma linha desocupada com o Discador BOL!
http://www.bol.com.br/discador
Ainda não tem AcessoBOL? Assine já! http://www.bol.com.br/acessobol


=
Instruções para entrar na lista, sair da lista e usar a lista em
http://www.mat.puc-rio.br/~nicolau/olimp/obm-l.html
O administrador desta lista é [EMAIL PROTECTED]
=



_
Converse com seus amigos online, faça o download grátis do MSN Messenger: 
http://messenger.msn.com.br

=
Instruções para entrar na lista, sair da lista e usar a lista em
http://www.mat.puc-rio.br/~nicolau/olimp/obm-l.html
O administrador desta lista é [EMAIL PROTECTED]
=



[obm-l] Re: [obm-l] questões básicas

2002-07-01 Por tôpico Paulo Santa Rita

Carissimo Jose Francisco,
Tudo Legal ?

1) Nao existe diferencas entre POSTULADO E AXIOMA. Os dois sao fatos que 
admitimos como evidentes e que, em determinado sistema formal, nao e 
passivel de demonstracao. E interessante registrar que aquilo que e AXIOMA 
ou POSTULADO em um sistema pode se tornar um TEOREMA em outro. A titulo de 
exemplificacao, considere os AXIOMAS de PEANO para a teoria do numeros. Em 
determinados sistemas eles sao AXIOMAS, em outros, TEOREMAS.

Historicamente, Euclides fazia ou ntendia a fazer uma diferenca entre 
POSTULADO E AXIOMA. Postulado era a verdade matematica evidente que nao 
precisava de demonstracao ( Ex : Por um ponto fora de uma reta dada so e 
possivel tracar uma unica reta paralela a reta dada ). Axioma era o 
PRINCIPIO DE PENSAMENTO evidente que nao precisava de demonstracao ( Ex : o 
todo e maior que qualquer de suas partes )

A diferenca que Euclides ensaiou adotar nao vingou e, em verdade, era um 
preludio das ligacoes da Matematica com a logica, coisa que ele nao tinha 
como abordar.

2) Em essencia nao ha diferenca : ambos sao afirmacoes sobre os objetos do 
sistema que precisam ser demonstradas. Todavia, na pratica, usa-se lema para 
designar uma VERDADE MENOR  que prepara ou antecede a demonstracao de uma 
verdade maior ou mais importante. Neste sentido, tomando um TEOREMA 
PRINCIPAL como origem conceitual, podemos dizer que o LEMA esta para o 
TEOREMA assim como o TEOREMA esta para o COROLARIO ( Nome bonito, nao ? 
Ainda vou ter um cachorro com esse nome )Um COROLARIO e uma consequencia 
direta e imediata do teorema, de demonstracao facil e que, sempre, vem apos 
o teorema.

3) DEPENDE ... Para a teoria dos numeros e um postulado ou axioma. Para a 
logica-matematica um teorema, para algumas construcoes um objeto.

4) Todas as quatro sao ramos da matematica. Ramos que se interpenetram e se 
influenciam. Talvez a topologia seja a mais fundamental de todas... Tao 
fundamental que so agora a gente comeca a entender como usar os argumentos 
topologicos de forma produtiva. NAO HA DEFINICAO FORMAL DE QUALQUER RAMO DA 
MATEMATICA ... Quer saber o que ele e ? Estude-o. Pratique-o. viva-o ! 
Existe uma enormidade de outros ramos !

5)Sim, existe. Em Matematica isso sao os objetos indefinidos. Por exemplo : 
na geometria, ponto, reta e plano. Na teoria dos conjuntos, o conceito de 
conjunto, na aritmetica, o conceito de numero.

Vale destacar o seguinte :

Num sistema formal nos temos os objetos indefinidos e as regras de 
construcao, com as quais podemos construir LEGALMENTE novos objetos. E como 
os POSTULADOS E AXIOMAS, que precisam ser manipulados PELAS REGRAS DE 
INFERENCIA para que possamos provar os teoremas.

ABRE PARENTESES :

Todas as suas interrogacoes se prendem ao ramo da Matematica chamado de 
Logica-Matematica. Um bom livro sobre este tema e :

O Teorema de Godel e a Hipotese do Continuo
Fundacao Calouste Gulbekian
Compilacao de Manoel Lourenco

E interessante ressaltar que a percepcao de um objeto ou de uma propriedade 
e uma atividade dinamica do intelecto ... A pessoa percebe os objetos e 
sente as leis, assim como vemos os objetos do mundo fisico e suas leis 
reciprocas. As provas e formalizacoes sao etapas posteriores do processo 
criativo.
A titulo de exemplificacao, Riemann, na dissertacao que lhe assegurou uma 
Catedra em Gottingen ( e que tinha Gauss com ouvinte ) enunciou varias leis 
e definiu vagamente varios objetos, sem apresentar uma unica prova ou 
definir formalmente qualquer dos objeto... Zero pra ele ? Nao ... Dez ! A 
conferencia em foco e considerada por muitos como a mais bela e importacao 
prelecao que ja ocorreu em toda a historia da Matematica. E ele ganhou a 
catedra em Gotingen !

Imagina se ao inves de ser Gauss ou seus amigos que estavam assistindo a 
prelecao fossem algum formalista ou Prof Pitonisa que, EM NOME DA MATEMATICA 
( Que ironia ... ), so aceita aquilo que esta bonitinho prontinho, de terno 
e gravata, provado nos minimos detalhes !

Oh ! Sorvei, olhos meus, o que vos der a vida ... A copiosa beleza no 
Universo Difundida !

FECHA PARENTESES

Um Abraco
Paulo Santa Rita
2,1739,070702

From: Jose Francisco Guimaraes Costa [EMAIL PROTECTED]
Reply-To: [EMAIL PROTECTED]
To: obm-l [EMAIL PROTECTED]
Subject: [obm-l] questões básicas
Date: Mon, 1 Jul 2002 15:35:07 -0300

Aproveitando que ontem e hoje foram discutidas questões básicas neste 
forum,
coloco algumas perguntas.

1. Qual a diferença entre um postulado e um axioma? Se esta pergunta
equivale a qual a diferença entre um periquito e uma banana, isto é, se
são duas coisas totalmente diferentes, então qual a definição de postulado 
e
de axioma?

2. Qual a diferença entre um lema e um teorema? Idem.

3. O que é 1 + 1 = 2? Uma definição, uma propriedade, um axioma? Se a
resposta for um axioma, como é que os matemáticos viviam antes de ele ser
proposto?

4. Aritmética, álgebra, geometria, topologia são considerados ramos da
matemática? Qual a definição formal de cada